You are on page 1of 448
Combining the above results, LY yt BY; ~ox{E) a Subsitution of the above results into Eq, (20.4) gives where = 3.167,y=14 have been used. Substitution of y= 21.0 yields ¢=0.708 = 70.8%. 2.1: m,,4=8.00 g and charge = -3.20x10° C 3.20107 © a) m, <= 2.0% 10", ve -1.6%10" C 8.008 _ 93310" and“ =8.58x10%, 207 Tg D) yay = Ny 21.2: cuent = 20,000 C/s and ¢ =100 ss =10~ s Q- f-2.00€ n= — 2 = 1.2510". 1.60% 10 C 21.3: The mass is primarily protons and neutrons of m =1.67 x10 kg, so: n= 70.0 ke pada" 1.67x10" ke About one-half are protons, so 7, = 2.10 x10 =x, and the charge on the electrons is given by: Q = (1.60x 10 C)x (2.10 x10) = 3.35 x 10? C. =4,19x10" 21.4: Mass of gold= 17.7 g and the atomic weight of gold is 197 g/mol. So the aumber of atoms A, x mol = (6.02 x 107)» (ete, 41% 107, a) n,=79x541x10" = 4.27% 10" q=n, x1.60 x10? C= 6.83% 10°C b) n, =H, =4.27%10™. 1.80 mol = 1.80 x 6.02 x10” H atoms = 1.08 x10™ electrons charge=—1.08 x10" «1.60 x10 C=-1.73 x10° C. 21.6: First find the total charge on the spheres: 2 23 q= ane FP? = fare, 4.57 x10) 0.27 =1.43 x10 C And therefore, the total number of electrons required is n= qe =1.43x10"* C/1.60x10"° C=890. © Ane, 21.7: a) Using Coulomb’s Law for equal charges, we find: 2 F=020N=—- 4 35 g = 55x10 = 7. x10 C. Anz, (0.150 my b) When one charge is four times the other, we have: 2 F=0220 N= a =>q= V1.375 x10 C? =3.71x107 C 4nzy (0.150 my’ So one charge is 3.71 x10” C, and the other is 1.484 10° C. 21.8: a) The total number of electrons on each sphere equals the number of protons. 0.0250 ke 0.026982 ke/mol b) Fora force of 1.00 x 10* Nto act between the spheres, 2 F=10° N--£5 q= y4re_ (10* N) (0.08 my =8.43x10% C. 7 =n) =¢/e= 5.27 «10 ©) nf is7.27%10" of the total number. n =n, =13%N,x* =7.25 «10%, The force of gravity nmst equal the electric force. 1 ¢ oe 60x10? cP © Ane, (9.1110 kg)(9.8 m/: mg= = ss = 25.8 m? = r= 5.08 m. Ane, r 21.10: a) Rubbing the glass rod removes electrons from it, since it becomes positive. 7.80 nC = (7.50 10 C) (6.25 « 10! electrons/C) = 4.69 x 10! electrons (4.69 x10" electrons) (9.11 x10 kg/electron) = 4.27 x10 ke. The rods mass decreases by 4.2710 kg, b) The number of elecirons transferred is the same, but they are added to the mass of the plastic rod, which increases by 4.27 x 10° ke. 2.11: F, is in the +.x- direction, so F, must be in the — x-direction and g, is positive. Rak, ign elt "3 "a 4, = (0.0200/0.0400)'g,|= 0.750 nC 6 Wd: a) F= zi = 0.200N-= 1_(0:550%10" Chan Arey Arc, (0.30) = gq, =43.4x10% ©. b) F = 0.200 N, and is attractive. 21.13: Since the charges are equal in sign the force is repulsive and of magnitude: kq? 3.50 x 10% C)* at. 5010 CY Loran r Azz, (0.800 m) 21.14: We only need the y-components, and each charge contributes equally. _1_ Q.0x10* C)(4x10° ©) Ane, (0.500 my’ ‘Therefore, the total force is 2F = 0.35 N downward. Fa sin a =0.173 N (since sin a= 0.6). 21.15: F, and Fare both in the +x-direction. Re lee - 6.749x10°N, R= _lael 1.124 x10 N ie B F=F,4+F,=18%10°N, in the +x-direction. _ x10 N-m?/C*) (20. x10 €) (2.010% C) WAG: Foy (060m) = 0.100 N Fo, is equal and opposite to Fig Gx. 21.4), so (F, ) =-0.23N (Fy), =0.17N Overall: F,=-0.23N F,=0.100N+0.17 N=0.27N ‘The magnitude of the total force is (0.23 N)’ + ( the force, as measured from the +y axis is tan! 23 _ 49° 0.27 7 NY = 0.35 N. The direction of 2.17: Fyis in the +x —direction. F,= pHMtl_ oar, 50 F,,=4337N 2 fia F, = Fy, + Fy, and F, =-7.00N F,, =F, - F,,=-7.00N-337N =-1037N For F,, tobenegative, 5 must be on the x-axis. Ke F=kEA, sold 0.144 m, sox=-0.144 m kag] x x 21.18: The charge ¢, must be to the right of the origin; otherwise both ¢, and q, would exert forces in the +x direction. Calculating the magnitude of the two forces: p 1 am _ (9x10 N-m4/C%V3.00%10* C)6.00x10° © 4 Ansy 1h (0.200 m)* = 3.375 Nin the+ «direction. = (9x10° N-m?/C?) (3.00 x10 C) 00x10 ©) a a is _ 0.216 N-m? 2 fa in the —x direction 0.216 N-m? 375 N+ 7.00 N ris = 0.144 m to the right of the origin = 0.0208 m* = 21.19: F =F, + F, and F = F, + F, since they are acting in the same direction at y=- 0.400 mso, 1 Eg 1.50x10° C , 3.20x10° C F= a+ 5 (0.200 m)* ~ (0.400 my (5.00 x10° of )-2s9-10" N downward. 4.00x107 C r= 00x10? ©)| ———} Ane, (0.200 my 2.21: a) 1 4@ 1 29Qa b) F,=0,F, =2—_ —# __ sna 9a Dae % Amey (a? + x°) Ane, (a? +7” c) Atx=0,F,= — 242 in the + y direction. dns. a @d a 21.22: a) =2%9O pp 4 cos? = aye Ey Anzy (a? +x 21.23: 1 @¢ 1@¢ 1 ¢ b) F=—— 4 4 J2 —_£ = (14 2V2) — © atan angle of 45° below the ) Axe) 20 vee gL 6 2) QL . positive x-axis 3 1 1 G.00%10" ©) _ 439 NjC,down toward the particle. diay 7 4x, (0.250 m) 14@.,.[4 4a, 7 2.24: a) B= = 210" ©) 150m. b) £=12.00N/C = 21.25: Let +x-direction be to the right. Find a, : Yp, = 41.5010? m/s, v, =-1.50 x10? m/s,¢=2.65x10% 5,4, =? V, =% py tat Bives a, = —1.132% 10? m/s F.=ma, =-7.516x10° N F'is to the left (- x-direction), charge is positive, so F is to the left. B= F/g= (7.516 x10" N)/[(2) d.602 10" ©) ]-23.5N/C 21.26: (a) x = 4d? 22 2(4.50 m) (3.00 x 16° s)? _F_ma_(9.11%10™ kg) (1.00 x10 m/s”) -@ @. 16x10" C =5.69N/C =1.00x10" m/s* E The force is up, so the electric field must be downward since the electron is negative. (b) The electron’s acceleration is ~ 10 g, so gravity must be negligibly small compared to the electrical force. 2 (0.00145 kg) 9.8 m/s") _ 9 19,-10° C, sign ismegative. 650N/C 10” kg) (9.8 m/s*) 1.60x10"C 2.27: a) |g/E= mg > bygl- ng ee OO =1.02*107? N/C, upward. 8 mas: a) = 21 261.6010" 9) 1 oy 10" NYC. dna rx, (6.0010 m) 9 b) £ tog 1 60x10 O _ sisytol nye. rion re, v? Arce, (5.29 x10 m)* 21.29: a) ¢ =—-55.0% 10 C, and F is downward with magnitude 6.20107 N. Therefore, £ =F /q=1.13x107 N/C, upward. b) Ifa copper nucleus is placed at that point, it feels an upward force of magnitude F = gE =(29)-1.6% 10°C. 113x107 N/C=5.24x10 N, 21.30: a) The electric field of the Earth points toward the pround, soa NEGATIVE charge will hover above the surface. (60.0 kg) (9.8 m/s”) ~1S0N/C P 1 @.92c/% © 4z6, (100.00 my? too great for practical use. mg=qE > q= =-3.92.C. gp 4 =1.38 x10" N. The magnitude of the charge is ney 21.31: a) Passing between the charged plates the electron feels a force upward, and just misses the top plate. The distance it wavels in the y-direction is 0.005 m. Time of flight - a 72 1.25%10° s and initial y-velocity is zero. Now, y=Ypyt + Lat” so 0.005 m= 4a(1.25x10" s)’ => a= 6.4010" m/s’. Buralso ny B= Oso LEHOs LWP B/N _ 364 NIC, 140.10 @ b) Since the proton is more massive, it will accelerate less, and NOT hit the plates. To find the vertical displacement when it exits the plates, we use the kinematic equations again: zee WL 4.05510- 9)? = 2.7310 m c) As mention in b), the proton will not hit one of the plates because although the eleciric force felt by the proton is the same as the electron felt, a smaller acceleration results for the more miassive proton. d) The acceleration produced by the electric force is much greater than g; it is reasonable to ignore gravity. 21.32: a) s ° Nem? C2) i : z-4, _ 9x10? N-m [Oy Csooxio © _ ¢g813x10¢ N/OV} Ane or, (0.0400 my Nem? /C? 9 |B, |= 22 N-i/C*) G.00 «10 ©) 08x10 N/C ” (0.0300m)’ + (0.0400 m)? 8) _196,9° Thus The angle of £,, measured from the x-axis, is 180— tan “($22 E, =(1.080x10* N/C) Ci cos 126.9° + sin 126.9°) = (6.485 «10° N/C)? + (8.64% 10° N/C)} b) The resultant field is E, + E, = (- 6485 x10" N/C)i+ (—2.813%10' N/C4 8.64 10" N/C) 7 =(-6.485x 10° N/C)?— (1.95 «10° N/C) 7 21.33: Let +x be to the rightand + y be downward. Use the horizontal motion to find the time when the electron emerges from the field: X= Xp 0.0200 m, a, = 0, v;, = 1.60x10° m/s, XX =Vp,t + 4,2? gives t=1.25x 10 s v, =1.60x 10° m/s ¥— ¥q = 0.0050 m, vy, — 0, = 1.2510" 5, v, =? Voy +, yma 5 = tees, 8010" m/s vs vt tv) =1.79 10° m/s t34i a) B=-11N/CP+14N/ch, so = Cin? +04) =17.8N/C. @= tan (-14/11)=-51.8°, so @=128° counterclockwise from the x-axis vb) F=£ ¢s0 F=(17.8 N/C) (2.510? C)=4.45x 10° N, i) at 52° (repulsive) ii) at +128° (repulsive). 21.35: a) F, = mg = (9.11% 10™ kg) (9.8 m/s’) =8.93x 10 N.F =cE= (1.60 x10" €) (1.00 104 N/C)=1.60x10" N. Yes, ok to neglect F, because F, >> F, b) E=10! N/C= FB =1.6x 10" N= mg = m= 1.63% 10" kg = m=1.79%104 m,. c) No. The field is uniform. m © €.60%10 €) (50% 10% 5}? e 21 21.36: a) xo tat! ALE yp 5, g = 200.0160 mn) 0.57 x10 he) _yyg nyc. db) vary + at Ago 149 10 mms, m, 7 v2; v2; i+ i 2 2 21.38: a) E=614N/C, F =gE=982x1077 N. b) F=e?/4ne, (1.0x10")' =2.3x10° N. c) Part (b) >> Part (a), so the electron hardly notices the electric field. A person in the electric field should notice nothing if physiological effects are based solely on magnitude. 21.39: a) Let+« be east. E is west and q is negative, so F is east and the electron speeds up. F,4q|E= (1.60210 C) (1.50 V/m) =2.403 x10°N a, = F[m = (2.403 x10? N)/(9.109 x 10kg) = + 2.638 x10" m/s? Vy, = + 4.50 10% m/s, a, =+ 2.63810" m/s’, x— x, = 0.375 m, v, =? vi avi, + 2a,(x— xp) gives v, = 6.33% 10° m/s b) g>0soF is west and the proton slows down. Fo =-|g|E =-0.602 x10" C) 1.30 V/m) = 2.403x10" N a, =F, /m=(-2403 x10 N)/.673x 10" ke} = -1.436 x10° m/s” Vy, = +1.90x10' m/s, a, =—1.436«10' m/s*, x — x, = 0.375 m, vy, =? Vy = Vox + 2a, (x xy) gives v, =1.59 x10" m/s 21.40: Point charges g, (0.500 nC) and g, (8.00 nC) are separated by x= electric field is zero when Ey = Ey = = GE = gyi? = g.2- 9) = qui’ — 2g, 1.2) 41.2", > (@ — an? + 20.2)qin — .2)'g, = 0 or 7.57? 41.27, -0.72=0 y=+0.24,-04 7, = 0.24 is the point between. .20 m. The 21.41; Two positive charges, q, are on the x-axis a distance a from the origin. a) Halfway between them, E= 0. 1 4 @ api er} Se aes 1 g @ b) Atany position x, E = +—4_|, x> prALalty'p * delat G@=ael "7" wl a , x<-a 4ne,\(@+x) (@—x)° For graph, see below. 21.42: The point where the bwo fields cancel each other will have to be closer to the uvgalive Charge, Leause itis salle: Aly, it vaul’L be between tbe bwy, sive Lie bwu fields would then act in the same direction. We could use Coulomb’s law to calculate the actual values, but a simpler way is to note that the 8.00 nC charge is twice as large as the 4.00 nC Charge. The zero point will therefore have to be a factor of J2 farther from the 8.00 nC charge for the two fields to have equal magnitude. Calling x the distance from the ~4.00 nC charge: 1.20+x= 2x x=2.90m 21.43: a) Point charge g, (2.00 nC) is at the origin and g, (3.00 nC ) is at x= 0.800 m. iy At c= 0.200m,e-—1al , _El@el sas nricright, (0.200 my’ ~ (0.600 m) ii) Avx-120m,e-—l@l_, Hal _ peo ye tent (0.400 m)? 4.20 my Klal ,_klaal 2 2 (0.200 m)? 1.00 m) b) F=-eF i) F=16x10? C.575N/C=9.2x10" Nlefi, ii) F= iii) At x=—0.200m, B= =405N/C left. 1.610" C-269N/C=4.3x10" N right, iii) F =1.6x107" -405= 6.48 x10°’N right. 21.44: A positive and negative charge, of equal magnitude gare on the x-axis, a distance @ from the origin a) Halfway between them, Z= , to the left. » [x| B= (129.67 -510.37)N/C = E=526.5N/C and @=75,7° down from the x-axis. 0.2 _1 Mite csaule an, (025m 2(6.00 x10 o[ ee d) x=0, y=0.2m:E = 21.46: Calculate in vector form the electric field for each charge, and add them. 1 (600x107 C) é = 1507 N/C Ane, (0.6 my" EN / 0.6) + 0.8)7 |= 21.67 + 28.87N/C ; of aaar yi yf Te Come“ vi} i INS => E= 28.4)’ + 28.8)? =131.6N/C, at oan 4-26 up from —xaxis. 1 2(6.0x10° C Ane, (0.15 m)’ 21.47: a) At the origin, E =— =~48007 N/C. b) At x=0.3m, y=0: 3 1 -oraat ol ate aaa | =28 N/C. o Ata-t4im, y=-04m: Ez rol aa 4m)" (0.5m) 05 } >E= a 129.67 -164.57)N/C => E= a0on/c and @ = 232° clockwise from +x- axis. 1_ (6.00107 C) (BS) _ D x= Oy 02 MiB, =0,B=— 7 Cismy 0754 = _ 10377 N/C 4 . 18 21.48: For a long straight wire, B= 5 is = 15x10 C/m _y ogy, mer 2ne,(2.5N/C) 21.49: a) For a wire of length 2a centered at the origin and lying along the y-axis, the electric field is given by Eq. (21.10). neq xafx? fa? +1 b) For an infinite line of charge: meee Graphs of electric field versus position for both are shown below. 21.50: For a ting of charge, the electric field is given by Eq. (21.8). zl al £- ——__=* _ ) Ane, (x +a’)? Q=0.125%10" C, a=0.025 mand x=0.4 m=> # = 7.08 N/C. i so with D) Foosing =~ Fung =-@ E = (— 2.50% 10% C) (7.07 N/C)=1.75 1077 N. onsing ona 21.51; For a uniformly charged disk, the electric field is given by Eq. (21.11): 21.52: The earth’s electric field is 150 N/C, directly downward. So, B=150= 5-5 @ = 3002, = 2.66 x107 C/m’, and is negative. 0 21.53: For an infinite plane sheet, E is constant and is given by a= directed eq perpendicular to the surface. raf) 100cm yy 1m 0 B= 226 N/'C directed toward the surface. 2, 21.54: By superposition we can add the electric fields from two parallel sheets of charge. a) E= b) B=0. coy ce) E=2- =~, directed downward. Sy By 21.55: 21.56: The field appears like that of a point charge a long way from the disk and an infinite plane close to the disk’s center, The field is symmetrical on the right and left (not shown). 21.57: An infinite line of charge has a radial field in the plane through the wire, and. constant in the plane of the wire, mirror-imaged about the wire: Cross section through the wire: Plane of the wire: \ f a es So co |e as, 2 : fo se | Length of vector does not depend on angle. Length of vector gets shorter at points further away from wire. 21.58: a) Since field lines pass fiom positive charges and toward negative charges, we can deduce that the top charge is positive, middle is negative, and bottom is positive. b) The electric field is the smallest on the horizontal line through the middle charge, at two positions on either side where the field lines are least dense. Here the y- components of the field are cancelled between the positive charges and the negative charge cancels the x-componeat of the field from the two positive charges. 21.59: a) p= qd = (4.5 x10" C)(0.0031 m) =14 x10! C-m, in the direction from and towards ¢,. b) If £ is at 36.9%, and the torque r= pH sin d, then: zt 7.2x10?N-m ee oe NYE. psind § (1.4107 C-m)sin 36.9° I 21.60: a) d= p/q=(.9x10" C-m)/ (16x10 C)=556x10' m b) tux = PE =(8.9x10™ C-m)(6.0x10° N/C)=5.34 x10 Nem Maximum torque: Na O E P 21.61: a) Changing the orientation of a dipole from parallel to perpendicular yields: (pEc0s90° — pEcos0°)= + (5.0%10 C- m)(1.6 10° N/C)= sew ») Jar-sx10% jo r-— 26410 D___oagax. 2 31.38 x10" J/K) . P 5 6.17x10 Com UO! Faye (t)= Frapue (3.0010 mn)= 22200 Cg 9 l= GPa Fa .00%N07 m= 5 x10° N/C. The electric force F = gE = (1.60 x10" C)4.11x 10° N/C) = 6.58 x10 N and is toward the water molecule (negative x-direction). 5 1 1 _(y+d/2y'-(y- 4/2)" _ yd 21.8: 9) Tap Gad) Gad J CGF a JE _ g Qyd qd y PB. "aT atigae der Ge aa ee dae, (Vid? /4Y Dney (yd? /4) Onegy b) This alsn gives the enrrect expression far F, sine y appears in the full expression’s denominator squared, so the signs carry through correctly. 21.64: a) The magnitude of the field the due to each charge is kk gd | A Aner? Ane, \(d {2 +2°))° where d is the distance between the two charges. The x-components of the forces due to the two charges are equal and oppositely directed and so cancel each other. The two fields have equal y-components, so: Qf 1 . r=0n,-22)__1 ___ lying » Tey a red in where @ is the angle below the x-axis for both fields. a/2 Ya /ayr+ thus -_ Arey((d [2 +x° Pi? p-{22 41 Yap fee Ane, A (a /2) +37) fia/ay +x ‘The field is the — y directions. b) Atlarge x, x” >> (¢/2)’, so the relationship reduces to the approximations qd Ane, Eaizote ™ 21.65: Fy Fy Nie m €oP te 2 5 ‘The dipoles auract, F=F,+h,-0, Fy=R,+Fy=2h, b) <> <> <> Opposite charges are closest so the dipoles attract. 21.66: a) O=—O Oa The torque is zero when p is aligned either in the same direction as E or in the opposite directions b) The atable orientation is when B isaligned in the same dircctionas Z ce) 21.67: sin d= 1.50/2.00s08 =48.6° Opposite charges attract and like charges repel. F,= Fy + Fy, =0 s -s ; =r 6.00107 C)G0.0x107 ©) a aa © 1.194%10° N m F,, =-Fisin@ =-842.6 N F,, =-842.6 Nso F, =F, +F,, =-1680N ty {in the direction from the +5.00 - j«C charge toward the — 5,00 - eC charge). b) Bee pS 1st em B The »—components have zero moment armand therefore zero torque. F,,and_ F), both produce clockwise torques. F,, = F,cos@ = 743.1N r= 2(E,,)(0.0150 m)= 22.3 N-m, clockwise 21.68: a) Fy = +} —— 28 cose +} 225 sin “oa. of ey Hs so Fy-+ lL G.00nCV6.00nC)_4;, 1 _6.00nCV6.000C) 45 gh _O00nENG.00nC) 3 472, (.00+16.0) x10 m) 5 Anz, (9.00 416.0) x10") 5 => F,=+(8.64% 10 Nyi + (6.48 x10 N)j. Similarly for the force from the other charge: a 1.20%104 NF azz, 0.0300 my" 1.20% u ‘Therefore the two force components are: F.=8.64x10° NF, =6.48 x10 -12.0 x10? =—5.52x10°N b) Thus, F = /F? +B? = J(6.64%10" Ny’ + (5.52107 Nj’ =1.03x10" N, and the angle is @= arctan(F, /F,)= 32.6,below the x axis ra) F- _#@, 1 _@ 4ne, (a+ x) Ane, (a- >, = a- 25. .-(422...))= —¥4| - 4s, @ a dre the equation of a simple harmonic oscillator, so: @__ | ka mesa Vad b) If the charge was placed on the y-axis thete would be no restoring force if g and Qhad the same sign. It would move straight out from the origin along the y-axis, since the x-components of force would cancel. 21.70: Examining the forces: 2 F, =T'sin@— F, = Oand DF, =Fcos@ —mg = 0. ja So 82- F =’ But tan@stod*=™iod 2.71: a) b) Using the same force analysis as in problem 21.70, we find: g? =4neyd?mg tan and @=2-(1.2)sin25 > = y4ne,(2- (1.2)-sin25°Y' an.25°(0.015 ke\(9.80 m/s”) => g= 2.79 x10 C. c) From Problem 21.70, mg tan@ = =, iq? __ (89910? Nm? /C)(2.79 x10* Cy’ mg(2L) sin? 9 4(0.6m)*(0.015 kg)(9.8 m/s*)sin 78 Therefore tang = 2. Numerical solution of this transcendental equation leads to @=39.5°. sin? = # => tané= al 21.72: a) Free body diagram as in 21.71. Each charge still feels equal and opposite electric forces. b) T=mg/cos20° = 0.0834 N so F, =Tsin20°=0,0285 N= “22, (Note: ri 7, =2(0.500 m)sin20°= 0.342 m.) c) From part (b), gg. =3.71x10-" C”. d) The charges on the spheres are made equal by connecting them with a wire, but we stillhave F, = mg tan @ = 0.0453N = 1-2 where Q = 9%, But the separation 7, is Gao v3 known: 7 = 2(0.500 m) sin 30° = 0.500 m Hence: Q= 24% = f4negFr?e =1.12x10 C. This equation, along with that from part (b), gives us two equations in q.and ¢,.9, +4, =2.24 x10 Cand 9g, = 3.70 10“ C?. By elimination, substitution and after solving the resulting quadratic equation, we find: g, = 2.06 «10° Cand a =180x107 C. 21.73: a) 0.100 mol NaCl > m,,, = (0.100 mol)(22.99 g /mol) = 2.30 g = m, = (0.100 mol)(35.45 g/mol) = 3.55 g Also the number of ions is (0.100 mol)¥, = 6.02x 10? so the charge is: ¢ = (6.02 «1071.60 x10" C) = 9630 C. The force between two such charges is: 1 g?_ 1 9630)" 4rre, (0.0200 my b) a= F/m=(2.09%10" N)/ (3.55 x10" ke) = 5.8910" m/s*. c) With such a large force between them, it does not seem reasonable to think the sodium and chlorine ions could be separated in this way. = 2.09 x10" N. “Ten oP 4a, F e 3 2.74: a) F, = 4.010 N= 5x10 Paar) (-0.3 my’ (40.2 m)’ _4.0x10°N _ (1262 N/C) b) The force acts on the middle charge to the right. ¢) The force equals zeto if the two forces from the other charges cancel. Because of, the magnitude and size of the charges, this can only occur to the left of the negative fq, _ ky (0.300—x)? (0.200 —x)? from the origin. Solving for x we find: x =—1.76 m. The other value of x was between the two charges and is not allowed. % ne. charge q,. Then: Fy = Fy > where x is the distance 2 w.75:a) F-49060). _1 SF toward the lower the left charge. The other Ane, (E/ {2 Ane, 1 two forces are equal and. opposite. Oo c 7 b) The upper left charge and lower right charge have equal magnitude forces at right angles to each other, resulting in a total force of twice the force of one, directed toward the lower left charge. So, all the forces sum to: w= | eee ot) ¢ (no+2)n 2 dns (W2Ly ) met? @ q 2g 21.76: a) roa e ot): 4ney\(y—a) (ytay y ») £@)-7 Fae aly" +(1+2/yy?—2), Using the binomial expansion: 2 2 2 eit 12 Fy oe cecal} Tey Saat Ans, y v2 Ans, y Note that a point charge drops off like 2 anda dipole like 21.77: a) The field is all in the x-direction (the rest cancels). From the + charges: eh ge th Ane, 4a? * Ameya? tx? fo? 452 dee, (a? +x)?” ach + contributes this). From the — 2g: 1 24 1 3/2 Hj “zal oe 41 * me xo es +x wear 22). ‘Arey x Foe ast Be exed)e ze! Baal sforx >> a. ane, x >) Faw 7 Note that a point charge drops off like + and a dipole like x 20.0 ¢ 12.0 g/mol electrons = g = (10.0)17,(1.60 x107?C) = 0.963 x10° C. This much charge is placed at the earth’s poles (negative at north, positive at south), leading to a force: 1 2 1 (0.96308)? = _F 2 1 O98 ANE) 5.1910'N. Are, (2Rag,) 406, (1.27610) b) A positive charge at the equator of the same magnitude as above will feel a force in the south-to-north direction, perpendicular to the earth’s surface: 2 Foo 4 sin ase Arey (Rau) 14 (0,963x10°C)? 4ne, J2 1.276 x10'm)* 21.78: a) 20.0 gcarbon => 67 mol carbon => 6(1.67)=10.0 mol => F=2 1.4410 N. 21.79: a) With the mass of the book about 1.0 ke, most of which is protons and neutrons, we find: #protons =4(1.0 ke)/(1.67x10-” kg) = 3.0x10™. Thus the charge difference present if the electron’s charge was 99.999% of the proton’s is Ag = G.0x 10% (0.00001)0.6 x10" C)=480C. b) F=k(Ag)/r? = (480 Cy /(5.0 mm)? =8.3 x 105 N — repulsive. The acceleration a= F /m =(8.3%10° N)/(1 kg) = 8.310" m/s’. c) Thus even the slightest charge imbalance in matter would lead to explosive repulsion! 21.80: (a) 4 4 4 ee be bv ee x > at. # te (b+ x) F.,(on central charge)= 7 1 _# | 1 ol © 4nzy| (b- x) (b +x)" __@ (4xP-b-x) _ ¢ dbx “Ans, (b= x¥(b+x)) Any @—x)'(b+ x) For x <<, this expression becomes ay, 2 F, x4, = 4. x Direction is opposite to x. 7a bb Key ) EF mai atx= mes @e__{_@ _), dé mreb? e a= =2n f >f-—— ea LOLA TEV me .28 x10" Hz QL: a) m= p¥ = p(4ar*)=(8.9x10 kg/m’ y¢2)0.00x 10° my = 3.728 x 107 kg, n= m/M = (3.728% 10” kg)(63.546 x10 ke /mol) = 5.867 x10 mol N= nN, =3.5x10" atoms N_=(29)(3.5x 10") =1.015 10” electrons and protons Gui = EN, — (0.99900)eN, = (0.100 « 10°71.602 10°? CYI.015 x 10) =1.6C Fakl=k 60" ry?" 00m) © =2.3x10°N 21.82: First, the mass of the drop: m= pV = (1000 kn (ett 4 ori: Next, the time of flight: ¢ - D/-v ~ 0.02/20 — 0.00100 s and the acceleration : _ 2d _ 2.00107 m) = 600m/s*. a (0.001 sy" 14 10" keg. 1 d=-at?=> 2 So: (41x10 kg)(600 m/s”) =Fi/m=qE/m>q= E= a= F/m=4k /m=> q= mal 8.00x10" N/C =1.06x10"" C. De. m, j, + 2a,dy=vjsin? a+ Say |AY|= hw vim, sin? a 2eE =>h, 1 bd) Ay=%,t+ zor #=4,, when Ay=0 . 1 =0= TM SIN A+ 5 Aha t, fo 2Qysin a 80 fone —_ a, or _2ym, sina ig ye d= Vibes = _ cos @ sin a ) 5 2, : 27) in? o 8) ggg = (x10? ms) #67 10g) sin? 30° _ 6 4 gy 24.610" C500 N/C) 4 %10°m/'s)*(1.67 x10" kg) cos 30° sin 30° (1.610 C500 N/C) =2.89m 2.84: a) E=S0N/C {2 y 8x10" C. e [+t fa. 1.2 my? [+500 N/C- 1 Arey | (4.00107 ©) (0.6 my (4.00 x10? ©) (0.6m) al |. 1 a l4xey 1° 2 Amey ry b) E=-50N/C= -50 |g (2- Fs 21.85: E=12.0N/C= = @=0.2my (+ (50.0) - —k(16.0nC) | k(12.0nC) i “an Ae eecnn ae tenn G.00my’ ~ (8.00m)? — S.00m) 1.60x10* C 1.20x10° = z~ |= 47.31x10 C=473.1nC. 9.0m’ 64.00 m’ >= 2soni( 2+ 4 Lf Qa 21.86: a) On the x-axis: dE, = E,= a ie (at+ry Anéy 4 ala tr — xy 12/11 | ane =o. i Any a atr bjIf a+r=x, then E ee 11); © 4e, ala-a x)” c) For >> a, F=22 a —a/2y - = #2 a4 fxd nw ax ax x 7 1 92 Note that for >> a, r= —a- x.) Charge distribution looks like a point due, r? from far away. usta) dp=—4@ = BOY wis ap, =§ BEY aya a, = (xP +9") a(x’ +y*) ax + yh ~KOy dy ar Thus: a(x +y'y 1 oxy & 1% 1 a_i Azey a g(x + yy? ane, a taf? a? Ane, x(x? +07)? g,-1 2) a x ray b) F,=—qE, and F, =—gE, where Z, and £, ate given in (a). in, alces +y ye ©) Forx>>a, R= 2a_qsativyya xa, ac Looks dipole-like in y-direction F, = -1@ ne x Looks point-like along x-direction - 4 Q.; 21.88: a) From £g. (22.9), B = Amey xfx? + a? 2 =z) (9.00107 C) (129x106 N/OF. Ane, (2.5x107 m)y(2.5x 107 m)’ + (0.025 m (b) The electric field is less than that at the same distance from an infinite line of 1 Phe 22 1302108 N/C). This is because in the das, x Ans, x2a charge (E, approximation, the terms left off were nepative. & — > ne, xll+3)" £ ®— (Higher order terms). 2a igh ). c) Fora 1% difference, we need the next highest term in the expansion that was left off to be less than 0.01: 2 7 7 < 0.01 = x < ay 2(0.01) =0.025m/2(0.01) = x <0.35em. a 21.89: (a) From Eq, (22.9), B= to _sp Atty fo a 2 =#- —__ 0x0“) __(_78358 N/O. Are (0.100 m),/(0.100 m)’ + (0.025 my’ b) The electric field is less than that at the same distance from a point charge (8100 1@Q a ri N/C). Since E,_, =——*4|1-* 4... |= E.,,, (Higher oder terms). Ane x*\) 2x c) For a 1% difference, we need the next highest term in the expansion that was left off to be less than 0.01: 1 570.0 = xe aJ1/(2(0.01) = 0.025/1/0.02 > x« 0.177 m 21.90: (a) On the axis, pe [-(+)" 4.00 pC/1(0.025 m)' [+ (0.025 my? J") “Oe, | (x ey (0.0020my => £=106N/C, in the + x-direction. b) The electric field is less than that of an infinite sheet Z,, =-2-=115N/C. 2p ©) Finite disk electric field can be expanded using the binomial theorem since the 3 expansion terms are small: = E « i a -| So the difference between the 0 infinite sheet and finite disk goes like & ‘Thus: AE(x — 0.20 em) = 0.2/2.5 — 0.08 — 8% and AE(x— 0.40 em) 0.4/2.5 = 0.16 =16%. eye 2.91: (a) Asin 22.72: E=— -(S) 2. a x 4.00pC/x(0.025 my'|_ (0.025 my?) a SEP ey cee +1 SE 2p (0.200 my =0.89N/C in the +x-direction. b) x>> B B= d-R° / 2x? + 34/8" 9) EQ o RoR’ 0 or a ie Qe, 2x Arex? Arey c) The electric field of (a) is less than that of the point charge (0.90 N/C)since the correction term that was omitted was negative. d) From above x= 0.2m 22989) _ oora1%, For x= 0.1m Egy =343N/C Eng = 36N/C wo 2828 90470 5% 11.9% a) f= FC): [) flande= | Peddes [ Fodax= [FCM Ca)+ ff f@aee Now replace —2 with ys = [" fa@dde=[ FoO)+4 [ far = 2 £ fQx)dr. b) e(a)=-9(-x):[" eteide= fl etedde + [Peed =- [2-day + [} eek. Now replace—x with y: = J" e(widx=-[e(nid(v)+ [f etedex =0. c) The integrand in £, for Example 21.11 is odd, so £,=0. 21,93: a) The y-components of the electric field cancel, and the x-components from both charges, as given in problem 21.87 is: 1-2 1 x > 1-2 1 1 > ga telh peck ate “dns, a \y GPta dz, a ly Of 4a' Og 1 Qga ney ay Ify>>a, Fe (-d-2? /2y' +. b) If the point charge is now on the x-axis the two charged parts of the rods provide different forces, though still along the x-axis (see problem 21.86). F-ck --( 4 -2}iand P= 9 8 -- 1 (4 A i dxe, alx xta a 2 For x >> oP gh OA 0%s..j 21243.) ia 2p x % te, ax 21.94: The electric field in the x-direction cancels the left and right halves of the semicircle. The remaining y-component points in the negative y-direction. The charge per unit length of the semicircle is: Bul FO oy a = ae sin = PINE GE. a So, E,= sin 946 =” cosay!? — 2 2D. swine. a a xa 21.95: By symmetry, £, = E,. For E,, compared to problem 21.94, the integral over the angle is halved but the charge density doubles—giving the same result. Thus, 2H, 2kO gigas Fe ae a ma 21.96: NF =05 TF cosa=mgs T= 5 VF, =057 sina=22 5 p-_@ 2e 2s,sin a mg _ qo ae 2? Ses age cosa 2asina Dagmg = a=arctan| —£ Qeymg m_E _14xd0°N/C 21.97; E =10 = = — = = 1429 keg / C. Eg aoe 106.8 m/s") ef kg mol 6.02107 carbons 1.6x10 © 4 15.4912 Catbons b) 1429 <2, —: - 25 C12x10% ke mol excess excess e of wie @,chares = 21.98: a) E,=E,,and E, =2E yyy, =f 7-20» V2, 2 math ead b) Ifall edges of the square had equal charge, the electric fields would cancel by symmetry at the center of the square. 21.99: a) (p= _2: |_ leo] [es] 00200C/m? 0.0100C/nr 4 £0200 C/m? 2 Wey Dey 26, 26, 26 2 => E(P) _vuloreat = 5.65%10' N/C, in the —x-direction. eo ») ray! a|_|o| lol ,0.0200C/m* 0.0100 C/m’ | 0.0200 C/m? Qe, Dey Dey 2, 26, 26, 2 > (a) = 2.220 Cl 69 10° N/C, in the +x-ditection, ey ° real | lel, les] 6.0200C/m? | 0.0100C/m? | 0.0200C /m? 2ey Dey 2 26 De 2é 2 > 2¢y= O8SOCL ON Ao 6p 208 N/C, in the+ x-direction. eo 8 Fag=_ ai| Jo|_lo|_ ,0.0200C/m* 0.0100C/m* _ 0.0200 C/m* Qe, 26, Dey 265 2e5 26, 2 > H(5)= SNOOC LH 5 65,10" N/C, in the + x-direction. 0 4 fan? 21.100: Fat 2.0010 C/o 44.13.10" N/m. A 2e5 = 42.26%10" N/m —6.00x10 C/m? 2, (Note that “+” means toward the right, and “—” is toward the left.) =-3.39x107 N/m 21.101: By inspection the fields in the different regions are as shown below: 2.102: a) Q= do =n(R? - Ro b) Recall the electric field of a disk, Eq. (21.11): £2 -1/ f@/a'=i]s0 Boo 2 (ba) Ve Tha YR Jaa Pas 209-32 (a/ Jeo / eq ora el eae iat a= Has /ayry EAR...) 2 } x oy = 2 {£—* 7-2 [4 > 8) 2 Rg (a/R) <1. se fe , and sufficiently close means that 1 1 1 Fay 2{E-d homts pad Le 21.103: a) The four possible force diagrams are: Only the last picture can result in an electric field in the -x-direction. b) gq =-2.00 nC, g, = 4.00, C, andg, > 0. } Bedeet« a sing, 1 _4__sing, % 4az, (0.0400 m) 4zce, (0.0300 my 9 sing 9 2% = 3/5_27 F= ed Flom (a+ 2x)Ife/? — [inl +2x+ adltz””) = pe te (el Es) _ kg? ‘in (a4+Ly HE a 4h 42a)) PF a(a+2E) spank” deta) ke c) For a>>L:F= ze nf Otel z (21n(1+ 2L/a)—Int + 2L/a)) ore ((t-5+ (2B Jar £ a 2a a a a 22.1: a) ®=£- A= (14 NIC) (0.250 m’) cos 60° =1.75 Nm?/C. b) As long as the sheet is flat, its shape does not matter. ci) The maximum flux occurs at an angle ¢ = 0° between the normal and field. cii) The minimum flux occurs at an angle ¢ = 90° between the nomal and field. In parti), the paper is oriented to “capture” the most field lines whereas in ii) the area is oriented so that it “captures” no field lines. 22.2: a) b= E- A= EA cosé where A= Afi left), = (410° N/C)(0.1 m)’ cos(90— 369°) = -24 N-m?/C Ay, =+8 (top)®,, =—(4v 10° N/C) (0.1m)? cos 90° = 0 fig, = +] (tight), = +(4%10° N/C)(0.1 my cos(90°— 36.99) = +24 N-m*/C fig, = -K (bottom)®, = (4x10? N/C)(0.1 m)* cos 90°= 0 fig, = +f (front), =+(4%10° N/C)(0.1m)? 003 36.9 = 32 N-m?/C (back)b, =—(4x10* N/C) 0.1m)‘ cos36.9° =-32N-m*/C b) The total flux through the cube must be zero; any flux entering the cube must also leave it. 22.3: a) Given that # =-Bi+C}-Dé, © =F. edge lengih L, and fig --}= ®, - B- Afig —-CL?. fig, =+h > ®,=E- 4a, =-DL’. fis, =+7 > ©, =. Afi, =+CL?. =4DL’. =H => 0,=£- 4A, =-BL’. -i>0,=8. 44, =+BI? b) Total flux= 7", = 0 Id: = B.A = (75.0 N/C) (0.240 m*)cos 70° = 6.16 Nm’/C. 22, a) = 8 d= Tay (stoue* comes 9 71.10% Nin? /C. b) We would get the same flux as in (a) if the cylinder’s radius was made larger—ihe field lines must still pass through the surface. ¢) Ifthe length was increased to /= 0,800 m, the flux would increase by a factor of two: ®=5.42x10° Nm?/c. 22.6: a) Oy =g,/e9 = (4.0010 C)/ey = 452 Nm’/C. D) Bg, = qp/é = (7.80107 C)/eg = -881 Nm‘/C. ©) ®s, = (q+. q2)/e = (4.00 - 7.80) x10 C)/eg = 429 Nmi/c. d) Dy, = (G+ 4p)/e) = (4.004 2.40)x 107 C)/eg = 723 Nmv’/C. ©) ®y = +92 +4)/eo = (4.00 -7.80 +2.40) x10 C)/e, = 158 Nm?/C. £) All that maiters for Gauss’s law is the total amount of charge enclosed by the surface, not its distribution within the surface. 22.7: a) B= g/e, =(-3.60x10% C)/s, =-4.07x 10° Nim?/C. b) ®=9/ey > ¢ =P = 4) (780 Nm/C)= 6.90x10" C. c) No. All that matters is the total charge enclosed by the cube, not the details of where the charge is located. 22.8: a) No charge enclosed so © = 0 _ -6.00%107 C e 8.85%10? C?/Nm? b) = -678 Nm’/C. ) 22.9: a) Since # is uniform, the flux through aclosed surface must be zero. That is: © = 48. d4- 2-4] pa =0 = | pdV =0, But because we can choose any volume we want, p must be zero if the integral equals zero. b) If there is no charge in aregion of space, that does NOT mean that the electric field is uniform. Consider a closed volume close to, but not including, a point charge. The field diverges there, but there is no charge in that region. 22.10: a) If p >0 and uniform, then q inside any closed surface is greater than zero. >o>0> jE dA > 0 and so the electric field cannot be uniform, ie., since an arbitrary surface of our choice encloses a non-zero amount of charge, £ must depend on position. b) However, inside a small bubble of zero density within the material with density p , the field CAN be uniform. All that is important is that there be zero flux through the surface of the bubble (since it encloses no charge). (See Exercise 22.61.) WAL Dggscs = 9/ ey = (9-60 x10 C)/e, = 1.08 x 10° Nmn?/C. But the box is 80x10 Nm?/C. symmetrical, 50 for one side, the flux is: ©... = b) No change. Charge enclosed is the same. 22.12: Since the cube is empty, there is no net charge enclosed in it. The net flux, according to Gauss’s law, must be zero 22.13: ©, =O..4/6 The flux through the sphere depends only on the charge within the sphere. Quon: = 850 = 6,(360 N-m?/C) = 3,19 nC @_ 1 (250«10"C Wald: a) E(r = 0.450 m+ 0.1) =—-4 = = 7 Ane)? 4aey (0.550my" b) E inside of aconductor or else free charges would move under the influence of forces, violating our electrostatic assumptions (i.e., that charges aren’t moving). was:ajee lal, [1 fal. [7 G10 O_o Axe, + 4nz,|E| Yue, 614 N/C b) As long as we are outside the sphere, the charge enclosed is constant and the sphere acts like a point charge. =TA4 NIC. 22,16: a) = EA =q leq = g= 6, EA =, (140 %10° N/C) (0.0610 m?) = 7.56 x10 C. b) Double the surface area: g = ,(1.40% 10° N/C) (0.122 m?)=1.51%107 C. IT: B= qh 2 = q = AxegEr? = 4ne,(1150 N/C) (0.160 m)’ = 3.27 x10" C. So the number of electrons is: 2, = -2#22¢ = 2.04%10", 22.18: Draw a cylindrical Gaussian surface with the line of charge as its axis. The cylinder has radius 0.400 m and is 0,0200 m long. The electric field is then 840 N/C at every point on the cylindrical surface and directed perpendicular to the surface. Thus § Ed = (EVA inden) = EVQ07L) = (S40 NIC) (2x) (0.400 m) (0.0200 m)= 42.2 N-m“/C The field is parallel to the end caps of the cylinder, so for them fe -28=0. From Gauss’s law: a C N-n? = &y@, = (8.854 x10 aS (42.2) =3.74 x10 € 22.19: om Agr 2tyictm fa 040m 0.2000 Jy =+4.80}40%m a) At point a,£, and £, are in the + y direction (toward negative charge, away from positive charge). E, = C/22,[(4.80 x 10° C/m) /(0.200 m)] = 4.314 x 10° N/C Ey = (1/26; [(2.40 x 10° C/m) /(0.200 m)] = 2.157% 10° N/C E= 8, +E, =647%10° N/C, in the y-direction. b) At point b, E7 is in the + y-direction and E> is in the — y-direction. E, = A/2n6,[(4.80 10° C/m) /(0,600 m)] = 1.438 x 10° N/C Ey = (1/264 [2-80 10° C/m) /(0.200 my] = 2.157 10° N/C E=£)-E,=7.2x10" N/C, in the — y-direction 22.20: a) For points outside a uniform spherical charge distribution, all the charge can be considered to be concentrated at the center of the sphere. The field outside the sphere is thus inversely proportional to the square of the distance from the center. In this case: 0.200 cm 0.600 cm b) For points outside a long cylindrically symmetrical charge distribution, the field is identical to that of a long line of charge: A E =(480 wel } =53N/C ok © Qnegr” that is, inversely proportional to the distance from the axis of the cylinder. In this case 0.200 cm 0.600 cm ¢) The field of an infinite sheet of charge is E = /2s); i.e., itis independent of the distance from the sheet. Thus in this case E = 480 N/C. F = (480 wel: )- 160 N/C 22,21: Outside each sphere the electric field is the same as if all the charge of the sphere were at its center, and the point where we are to calculate £ is outside both spheres. £, and £, are both toward the sphere with negative charge. R= pla = eboelOe, = 2.591x10° N/C i (0.250 my @|_ ,3.80x10°C ny (0.250m)? E= E, + E,=8.06%10° N/C, toward the negatively charged sphere. S.A71%10° N/C 22.22: For points outside the sphere, the field is identical to that of a point charge of the same total magnitude located at the center of the sphere. The total charge is given by charge density x volume: q=(7.50 acfmyGxy0.150m)! =1.60x10°°C a) The field just outside the sphere is ° Nem IC? 10 po, AIP Nem C4 0.06210 ©) _ ay 4 nye (0.150 my © dre, b) Atadistance of 0.300 m from the center (double the sphere’s radius) the field will be 1/4 as strong: 10.6 N/C ¢) Inside the sphere, only the charge inside the radius in question affects the field. In this case, since the radius is half the sphere’s radius, 1/8 of the total charge contributes to the field: pew 2X10? Nm? / C7) 1/8) 1.0610" C) _ 21.2N/C (0.075 m)” i 22,23: The point is inside the sphere, so E = AQr/ R? Example 22.9) EK _ (OU N/L) (0.220 my" 2 (0.100 m) =10.2nC 22.24: a) Positive charge is attracted to the inner surface of the conductor by the charge in the cavity. Its magnitude is the same as the cavity charge: ¢,,,, = +6.00 nC, since E =0 inside a conductor. b) On the outer surface the charge is a combination of the net charge on the conductor and the charge “left behind” when the +6.00 nC moved to the inner surface: Gor = Ymce + Youre: => Foner = Fut —~Finuee = 5-00 nC ~ 6.00 n= =1.00 nC: 22.25: S, and $, enclose no charge, so the flux is zero, and elecitic field outside the plates is zero. For between the plates, S, shows that: Ed =q/2,=0 4/e) => E=a/ey. 22.26: a) Ata distance of 0.1 mm from the center, the sheet appears “infinite,” so: aa fé di=524-4>54-—4 = 15010" C een NC. 2 Degd — 2e9(0.800 my b) Ata distance of 100 m from the center, the sheet looks like a point, so: 1 (7.50x10° © Ane, ne, (100m)? c) There would be no difference if the sheet was a conductor. The charge would automatically spread out evenly over both faces, giving it half the charge density on any as the insulator (o :). E, === 2+ near one face. Unlike a conductor, the insulator ds the ig i 675x107 N/C. charge deusity in some sense. Thus one shouldn’t think of the charge as “spreading over each face” for an insulator. Far away, they both look like points with the same charge. 22.27: a) b) §E-dA= 8QnrL)= Q_ £5 &s Yq c) But from (a), .=o2rR, so E=72>, same as an infinite line of charge. 22.28: All the o's are absolute values. @ d 4hp- S212 = 1 Eyota ta—ai) -6 #C/m? +2 pC/m? +4 wC/m’ —6 pC/m*) 0 =2.82x10° N/C to theleft ) i 1% = yy CuCl +2 nC? +4 ucla? 8 nC fat) = 3.95% 10° N/C totheleft. ©) =e pC/m? +2 pC/m —4 2C/m’—6 2C/m’) 0 =1.69x10° N/C to the left 22.29: a) Gauss’s law says +0 on inner surface, so £ = 0 inside metal. b) The outside surface of the sphere is grounded, sono excess charge. c) Consider a Gaussian sphere with the—Q charge at its center and radius less than the inner radius of the metal. This sphere encloses net charge —Q so there is an electric field flux through it; there is electric field in the cavity. d) Inan electrostatic situation E =0 inside a conductor. A Gaussian sphere with the — Q charge at its center and radius greater than the outer radius of the metal encloses zero net charge (the — Q charge and the + Q on the inner surface of the metal) so there is no flux through it and £ = 0 outside the metal. e) No, E=0 there. Yes, the charge has been shielded by the grounded conductor. There is nothing like positive and negative mass (the gravity force is always attractive), 80 this cannot be done for gravity. 22.30: Given B = (-5.00 (N/C)-m)xé + (3.00 (N/C)-m)zk, edge length £=0.300m, Z = 0.300 mand 4, =- j= ®,=£-A,4=0. Aig, = + k= ©, = E+ fiy A= (3.00 (N/C)- m0)(0.300 m)*z = (0.27 (N/C)m)z= (0.27 (N/C)m)(0.300 m) = 0.081 (N/C) m2. fig, By = E-fiA= 0. fig, =-h=> ©, = B-g A=-(0.27 (N/C)- m)z=0(z= 0). fi, =+2 = ©, = B-Ai,, A= (-5.00 (N/C)-m)(0.300 m)?x = -(0.45(N/C)- mx = (0.45 (NC) m)(0.300 m)— (0.135 (N/C) m’). fig, =f =O, = B fig, A= + (0.45 (N/C)- mx = 0 @=0). b) Total flux: =O, + , = (0.081— 0.135) (N/C)-m? =-0.054 Nm’ /C @ =-4.78 x10 BC 22.31: a) a b) Imagine a charge g at the center of a cube of edge length 2L. Then: ®=@/e,. Here the square is one 24th of the surface area of the imaginary cube, so it intercepts 1/24 of the flux. That is, ® = g/24e,. 22,32: a) D= E4 = (125 N/C)(6.0m’)=750N-m'/c. b) Since the field is parallel to the surface, = 0. c) Choose the Gaussian surface to equal the volume’s surface. Then: 750 — Edm aay = E= ahs (2.40 x 10 C/g, + 750) =577 N/C, in the positive x-direction. Since ¢ <0 we must have some net flux flowing iz so EA > -| EA | on second face. d) ¢< 0 but we have E pointing away from face J. This is due to an external field that does not affect the flux but affects the value of E. 22.33: To find the charge enclosed, we need the flux through the parallelepiped: ®, = AE, cos 60° = (0.0500 m)(0.0500 m)(2.50 x 10* N/C) cos 60° = 37.5 N-m?/C ®, = AE, cos 120° = (0.0500 m)(0.0600 m)(7.00 x 10* N/C) cos 60° = —105 N-m?/C So the total flux is =, +, = G7.5-105) N-m‘/C =-67.5N- m?/C,and g=@s, =(-67.5 N-m’/C)s, =-5.97 x10 C. b) There must be a net charge (negative) in the parallelepiped since there is a net flux flowing into the surface. Also, there must be an external field or all lines would point toward the slab. 22.34: The a particle feels no force where the net electric field is zero. The fields can cancel only in regions A and B. Ene = Exact Ad Dnsyr 2s, p22 =— 22 0.16 =16em (100 Clim?) The fields cancel 16cm from the line in regions A and B. 22,35: aio ncn The electric field &, of the sheet of charge is toward the sheet, so the electric field E, of the sphere nmst be away from the sheet, This is true above the center of the sphere, Let r be the distance above the center of the sphere for the point where the electric field is zero. B=, 69 See J Or 2 Ane, R 2no,R* 2x(8.00x10" C’m’*)(0.120 my" Q 0.90010 C 22.36: a) For r , £=;4-%, since again the total charge enclosed is +g. b) Ee c) Charge on inner shell surface is ~9. d) Charge on outer shell surface is +g. e) 22.37: a) r < R, E =0, since no charge is enclosed. b) R2R, E = -¥. charge enclosed is 20. Fagor? since we 22,38: a) rb,E=-2 =i , since the total enclosed charge is 20. b) The surface charge density on inner surface: o =—72-. c) The surface charge density on the outer surface: ¢ =—=2, a0 d) e) 22.39: a)(i) rd, B= 7%, since Q=+ 6g. b)Ci) small shell inner: Q (ii) small shell outer: Q (iii) large shell inner: Q (iv) large shell outer: Q =+6g 22.40: a)(i) r d, £=0, since the net charge enclosed is zero. E b)(i) small shell inner: (ii) small shell outer: iii) large shell inner: (iv) large shell outer: 22.41: a)(i) r d,£=— 2, since charge enclosed is — 2g. Fee b)(i) small shell inner: Q=0 (ii) small shell ower: Q=+ 2g (iii) large shell inner: Q =~ 2g (iv) large shell outer: Q=-2g 22.42: a) We need: Az p 7 — 280 pR? 32. -Q=——((2RY - 8°) => Q =———_ 3 p= — Q 3 (QRY-R)30 3 P= FR b) r2R, £=0, since the net charges are zero. RercOR, O= hdr 2063 py pe 2 a Pty 3 Aner’ Bor Substituting p fiom (a) B= 2-2 - c) We see a discontinuity in going from the conducting sphere to the insulator due to the thin surface charge of the conducting sphere—but we see a smooth transition from the uniform insulator to the outside. \ 22.43: a) The sphere acts as a point charge on an external charge, so F = gE =;1%, radially inward, Teg (b) If the point charge was inside the sphere (where there is no electric field) it would feel zero force. 22.44: a) ededaeges attire} om \ bw + Edn" em OP 1 rag Poa 2, @-f)_ « ¢ ) 2 r 4mm, (B— a") (ii) b Bar’ i # % 4negr" (iv) cc,#=74-2, radially outward, since again the charge enclosed is the same as in part (a). 22.45:a) a Aimer =—As ANE Reger = Pe sa) @) rb, EQart)=1=5" = p=. & % E07 b) (@ Inner charge per unit length is —@. (ii) Outer charge per length is + 2a. 22.47: a) (i) E=B (iii) > b, there is no net charge enclosed, so the electric field is zero. \ es b) (i) Inner charge per unit length is — a. (ii) Outer charge per length is ZERO. 22.48: a) + < R, EQanrl)=* b) r> Rand X= paR’, EQeri)=2 =! Fam as leg Tae or => B=, radially outward. c) r= R.the electric field for BOTH regions is E = 2, so they are consistent. dy k 22.49; a) The conductor has the surface charge density on BOTH sides, so it has twice the enclosed charge and twice the electric field. b) We have a conductor with surface charge density o on both sides. Thus the electric field outside the plate is © = E(2.A) = (20d)/e, > E =o/e,. To find the field inside the conductor use a Gaussian surface that has one face inside the conductor, and one outside. ‘Then: = By A+ Ed = (cA) e) but Boy — o/s) > Eyd- 0 B,-0. 22.50: a) If the nucleus is a uniform positively charged sphere, itis only atits very center where forces on a charge would balance or cancel = Fag? =£ (5) =. ey \R 4negR = F=qh-- ler. Anz, RB So from the simple harmonic motion equation: a 1 er 1 é 1f1 é F=-mo'r=-—_* a= 5 = f=—,|——. Ane, B Arce, mR Ix \ Ane, mR z o) If f=4.57x10" Hz [1 QarY dns mR =3.13x107 m wl erat!” rucapsca d) If y > R then the electron would still oscillate but not undergo simple harmonic motion, because for r > R, Foc 1/r” , andis not linear. 22.51: The electrons are separated by a distance 2d, and the amount of the positive nucleus’s charge that is within radius d is all that exerts a force on the electron. So: ke® = = Byung =the? £3 d= BB d= RID. 2 gyt — Fa 7 22.52:a) O(r)= O- fear = o-& Hf 21% 2 sin dO dep = 02x etre ay ioe QE Gee Note if r > 2 Ces 30. = (26% — abr? — dar — 2) = Qe™"*[2(r/ a)? + 2r/ ay) + 1. b) The electric field is radially outward, and has magnitude: (2G aJ° + 2(r/a) +1). 22.53: a) At r=2R, F =9,E = qh 2m = bmn" _ 94 NY Tey da? FG Ard aI o: a=Fi/m= 11x =10x ms". Ss Fim =94 N/9.11 x10" 1.0 x10" mis’ b) Atr=R, a= 4a, =4.1%10" ms’. c) At r= R/2,Q =+(82e) (¢ because the charge enclosed goes like r*) so with the radius decreasing by 2, the acceleration fiom the change in radius goes up by (2)? = 4, =2.1%10" m/s’, but the charge decreased by 8,30 a =42,, d) At r=0,Q=0,s0 F =0. 22.54: a) The electric field of the slab must be zero by symmetry. There is no preferred direction in the 3 -z plane, so the electric field can only point in the x-direction. But at the origin in the x-direction, neither the positive nor negative directions should be singled out as special, and so the field must be zero. b) Use a Gaussian surface that has one face of area A on in the y -z plane at x=, and the other face at a general value x, Then: xéd:0— RA = Leet _ PAY &o a with ditection given by *, Note that E is zero at x=0 Now outside the slab, the enclosed charge is constant with x: xzd:0=f4— Ge _ PA , ppd & fo again with direction given by 4 22.55: a) Again, E is zero at x = 0, by symmetry arguments. Pode, py _ Bok” by xsd: = 4 = Gent = Pol ja? ae 5 * Bad eg |x a4 ~ direction. eed | 4 x2d:0= By = Sam _ Pod [eta 2 Ea Pe vin ditection, ee a ° Be |x 22.56: a) We could place two charges + on either side of the charge +¢: b) In order for the charge to be stable, the electric field ina neighborhood around it must always point back to the equilibrium position. c) If q is moved to infinity and we require there to be an elecitic field always pointing in to the region where ¢ had been, we could draw a small Gaussian surface there. We would find that we need a negative flux into the surface. That is, there has to be anegative charge in that region. However, there is none, and so we cannot get such a stable equilibrium. d) For anegative charge to be in stable equilibrium, we need the electric field to always point away from the charge position. The argument in (c) carries through again, this time infewing that a positive charge must be in the space where the negative charge was if stable equilibrium is to be attained. 22.57: a) The total charge: g = 47 { Pril-r/ Ryde = all r'dr -f P/Rér] a 3 9 4nR py AnR? 30 12 12 aR* = ¢ = Amp, [R'/3- R8/4] = 12 b) r 28, all the charge Q is enclosed, and: = E(4ar*) = Qi, > E= 7-4, the same as a point charge. c) r 2 - oe, Or, <2 R80 Bigg -iea- = 42 or 3 3R 3R 22.58: a) “ ‘ dr * A oe =4 Pedy = Ay 1- | ar = 2dr -—[" Pai O= te] ployer rl: =): 1 wolf rape ] Ra RK Azp,| —-—:— =>O=0 | BR “| g by r2R,f B-di-Se1-05 8-0 £0 z.4- Nplate 4 7 ae 4 Be ) rsR GE weet oleae! => Egy? = . if wide rar! d) e) Ey fo _ rote 9, 8 or Bey 3e,R a{r-Z)-8 yet] a oR 2) By 2 2] 12% 22,50: a) ©, = 43-44 Gm pS eB nm, 7 b) For any closed surface, mass OUTSIDE the surface contributes zero to the flux passing through the surface. Thus the formula above holds for any situation where m is the mass enclosed by the Gaussian surface. That is, =4 g-d4=—406M,,, 22.60: a) ®, = g4er? = AnGM = g = —S!, which is the same as for a point mass. : b) Inside ahollow shell, the Af.,., = 0,80 g=0. ¢) Inside a uniform spherical mass: e GMr ©, = gdm? =-40GM = tna (a 5)>8-- 9 which is linear in r. 22.61: a) For a sphere NOT at the coordinate origin: s i P=P-6 50 =4r? p= Or Aw por & & 3 Bey in the #’- direction. =p 2@-4) Beg b) The eleciric field inside a hole in a charged insulating sphere is: jar BP 6) pb Eon = E stam Eg Note that £ is uniform. 22.62: Using the technique of 22.61, we first find the field of a cylinder off-axis, then the electric field in a hole in a cylinder is the difference between two electric fields—that of a solid cylinder on-axis, and one off-axis. -b 5 ©=2Inrl E= Dart = fin? eo &y ~ 7 pb) pb Rican: By.= 0 - PLE —5) _ 2 Note that Fis uniform. De, Dey Ie 22.63: a) x= 0: no field contribution from the sphere centered at the origin, and the other sphere produces a point-like field: Beaty) 8 1 Og dm, ORY 4a, AR b) x=R/ 2: the sphere at the origin provides the field ofa point charge of charge @=Q//8 since only one-eighth of the charge’s volume is included. $o: 1_/ @/8) Q 7) 2 1 Os E@=R/2=-7— — 2 7-+ Lan -ainf -— Si. Geek?) 1 (& DF GR/ DO) dns, RP = na, TBR? : the two electric fields cancel, so F =0. R: now both spheres contribute fields pointing to the right: ©) x= d) x= 1 1005 dae, OR 21g GRY?) Ba=3R-2 5 22.64: (See Problem 22.63 with Q > —Q for terms associated with right sphere) 1 0; E (x=0)=4+— a) E@nO=4 5 seal z(,-8).[@/), © L{Q 40}; maleate " l e* | 3 E(@=R)= te Q 9);_3 2128) 1 80 ; i GRY R* Arey |9R? RR? Arey 9R® 22.65: a) The charge enclosed: 3 2 0=0,+ 0), where Q, = oAZR2Y _ aR d) £(@=3R)= BA, and Q, = 4x(20) |"? — FP Rar 3.0~C‘(S a lan & Sige (R= R/S) _ (RY R*/16)) _ ManR? 3 4k 34 _ 1aaR* 8g. On OER Z b) 7 W =-AU=- 0.357). 23.2: W =-1.9x10°)=— AU =U, -U, 3U, =1.9%10° + $.4x10°I = 73x10) 23.3: a) E=K,+U, = 5 (0.0015 kg}(22.0.m/5)*+ 6 6 K(280%10* C7.S0X10°C) _ 9 6g y 0.800 m BB, ~ jm} 4 Miley, ~ a b) Atthe closest point, the velocity is zero: 6 6 sneer < EE opp RO UOT OKTEOWI OO) pay seu, r 0.608 J 6 s wear U = 0.400) = B22 p= SHO 30X1TONT AOC) _ 575m = 0.400 J 6 6 we 9 pT ee _ewey r 0.250 m b) @) K,=K,+U,-U, = 03+ &(4,60x10*C) (1.20x10°C) = 0.0994 025m 0.5m 1 > K, = 0.09941 =F my} vy 380x107 kg 26.6 m/s. (i) K, = 0.1895, v, =36.7 m/s. Gii) KK, = 0.198, v, =37.6m/s. 2 2 wae: U = 2 6g? = 6x. 210° Cy? = 0.078. 0.500m 0.500 m (4.00 nC\(-3.00 nC) , (4.00 nC)(2.00 nC) We , Whe) _ (0.200 my (0.100m) ls Ms (23.00NE)@2:00.06) (0.100 my =-3.60x107 J. yeu = 0.0-1{ 4 2% | So solving for x we find: ‘2 tien => 60x" — 26x +1.6=0=> x= 0.074 m, 0.360m. Therefore x 2 — x = 0.074 msince it is the only value between the two charges. 23.8: From Example 23.1, the initial energy , can be calculated: E,=K,+U, =50 11x10™'kg)(3.00x10' m/s)’ 4 ML 160%10"" Cy3.20210"" C) 10m => E,=-5.09x10? J. When velocity equals zero, all energy is electric potential energy, so: 2 =s.09107° 7-8 =,» 9.06.10 m r 23.9: Since the work done is zero, the sum of the work to bring in the two equal charges q must equal the work done in bringing in charge Q. kg’ _ 2kgQ g W, =Wy> a a as QEa 23.10: The work is the potential energy of the combination. UsU,,+U,, +U, he(2e) _ ,_ke(—e)_, k(e)(2e) oe “5J2x10%m 5x10" m 5x10 m ke? 2 “smal Bo) _ 9.010? Nm’ /C?) (1.610 Cf 5x10 m 73x10? J Since Uis negative, we wantdo + 7.31x 10°" J to separate the particles K,4U,; K,=U,=0s0 K,=U, 2 ua <_(2+242) 1 5e with r=8.00x10- m tale © © Arey + U, =144x 10" J=9.00eV 2 23.12: Get closest distance y. Energy conservation: > my? + gm =— ? 2 © Nm? /C? -19 cya ya fe. Gxt? Nav Ie 30.6107 CF 36.10 m my 1.67 x10™ ke)(10" m/s) Maximum force: ke _ 9x10? Nm?/C’) 1.6 x10 Cy? 38x10 m)* = 0.012 N 23.13: K,+U,=K,+U, U=q¥,30K,+q¥,=K, +4¥, K,=K4+q¥4—V,)= 0.00250 J + (- 5.00 10% C) (200V —800 V) = 0.00550 J v, = 2K, /m =7.42 m/s IL is faster at B; a negative charge gains speed when it moves to higher potential. 23.14: Taking the origin at the center of the square, the symmetry means that the potential is the same at the two comers not occupied by the + 5.00 C charges (The work done in moving to either comer from infinity is the same), But this also means that no net work is done is moving from one comer to the other. 23.15: £ points from high potential to low potential, so V, >, and V, <¥,. The force on a positive test charge is east, so no work is done on it by the electric force when it moves due south (the force and displacement are perpendicular); V, = ¥,. 23.16: a) Wf =— AU =qEd =AK =1.50x10° J. b) The initial point was at a higher potential than the latter since any positive charge, when free to move, will move from greater to lesser potential. AV =AU/q= (1.50 10% J/ (4.20 nC) = 357 V. 1.50*10% J ©) gd =1.50x10" Ja B=" _=5.95x10° NIC. (4.20n0)(0.06 m) 23.17: a) Work done is zero since the motion is along an equipotential, perpendicular to the electric field. b) W=qEd =(28.0 no 4.00.10! ~\cao% m)=7.5x107 J m c) W=qEd = (28.0 no 4.00 x10" > a,¥=——2, just like a point charge of charge + 2g. tre, x 23.23: a) by y= MoO 29, ror c) The potential along the x-axis is always zero, soa graph would be flat. d) If the two charges are interchanged, then the results of (b) and (c) still hold, The potential is zero fq tg (aty) (a-y) 23.24: a) |y|>a:¥ x — ax SO ~umenemecaenne potential of a point charge x -q. (Note: The two charges must be added with the correct sign.) kg 2k 1 2 23.26:a) ¥=—2 ~“# = pg) — -__+__ | inl lio fata be b) ¥=0, when y=" 25 3) =a! y= c) Sealed 290 aww ia the pomnil of a poiancharge=y. y i 23.27: W =—AU =—¥q = (295 V) 0.6010"? C)=4.72%10"" J.But also: 01x10" mys. 93.98: a) Baad = +8 _oaism d E 120N/C b) VB gg CACM @230,10" c k k c) The electric field is directed away from ¢ since it is a positive charge. 23.29: a) Point b has a higher potential since it is “upstream” from where the positive charge moves. V,—¥, = E(@-a)=-|E|(-a)>¥, -¥, = |E|@—a)>0 V_240V b) E-— =~ s00N/C. @ 03m c) W=-AU=- AV =- (-0.20x10* C\- 240 V)=-4.8 «107 23.30:(a) V =V, +V¥oq > 0, so Vis zero nowhere except for infinitely far from the charges. Q 20 o<—d—+e x The fields can cancel only between the charges EE (d= 3)? = 2x? ¥ can be zero in 2 places, A and B. K@2) 94 5=4/3 x at B:AC@) , FCO) 9 yg dt+y Ey = Eyg to theleft of — Q. © Note that £ and V are not zero at the same places. 23.31: a) K,+q¥,=K,+qh; a —-Wy=Ko-Ky = 4.099x10" 1.602 x10" C bmi = 2.915 x10" J The electron gains kinetic energy when it moves to higher potential. b) Now K,=2915x10" J, K,=0 Ky, = Ki ¥A-V,= =+182V The electron loses kinetic energy when it moves to lower potential x107 C 23.32: a) Va = BB 0x10" O) _ es gy, r 0.48 m @ 9 ce eee 3.50 x10 ©) _ispay 0.240 m c) Since the sphere is metal, its interior is an equipotential, and so the potential inside is 131.3 V. 23.33: a) The electron will exhibit simple harmonic motion for x << a, but will otherwise oscillate between +30.0 cm. b) From Example 23.11, v kQ yrta = AV = ot a 1 = AV =k24.0x10° ©) | ——— - ——__—_______ 0.150m (0.300 m)* + (0.150 my* =796V Bur W == qa¥= Lm? >y =1.67x10" m/s. 23.34: Energy is conserved: 2 2 Lin? = qAV = Ay = 0-572 107 ke) A500 ns)" _ 9 o117V, 2 21.60% 10" C) But: X Ing/r) > 4 =r exp (=) eee ow Ty » AV= 2 2ree9(0.0117 V) = r= (0.180 m)exp ( S000 Gm Jessi Vv 3607 @ 0.0450m b) F = Eq = (8000 N/C) (2.4010 C)=1.92 10° N. c) W= Fd = (1.92 10° N) (0.0450 m) =8.64 «107 J, d) AU=A¥q =(— 360 V) (2.4010? C)=- 8.64107 J. 23.35: a) E = =8000 N/C. 23.36: a) V = Ed = (480 N/C) (3.8107 m)=18.2 V. b) The higher potential is at the positive sheet. ¢) B=2 59 =6,(480 N/C)=4.25 x10" C/m’. £5 23.372) e-Esa-Fe ATSON = 158 «107 E 3.00x10° V/m b) B= 0 =6,(3.00x10° V/m) =2.66 x10" C/m’. fe a _470x10° C Se b) V = Ed = (S311 N/C) (0.0220 m)=117 V. c) The electric field stays the same if the separation of the plates doubles, while the potential between the plates doubles. 23.38:a) E =S311 N/C. 23.39: a) The electric field outside the shell is the same as for a point charge at the center of the shell, so the potential outside the shell is the same as for a point charge: V=—*_forr>R. A: 0597 ‘The electric field is zero inside the shell, so no work is done on a test charge as it moves inside the shell and all points inside the shell are at the same potential as the @ Eg surface of the shell: ¥ = for r C= 240 V) (0.0130 m)~ = 7.85 x10* V/m**. b) = -= = -Fa =- + (1.85%10* V/m"")x!? = ( 1.05 x 10° ee? vim, mn toward cathode. c) F =—eF = (1.05 x 10°) (0.00650)! V/m) (1.60 x10" C)=3.14x10-" N, toward anode. 23.52: From Problem 22.51, the electric field of a sphere with radius R and gq distributed. uniformly over its volume is £ =—""_. for r < Rand E=—4 > torr 2R 4K Ep. bregy -¥,=[° Har. Take bat infinity and V/, = 0. Letpoint a be adistance r < from the center of the sphere. gr @ @ v= drt ar=—4_|3- I die 8 Aye ( =] Set q = + 2e to get V, for the sphere. The work done by the attractive force of the sphere when one electron is removed from y =d to ois 2° a Wise =— eV, = 3-4 oe Sak ( e } ‘The total work done by the attractive force of the sphere when both elecizons are removed is twice this, 217,,,.... The work done by the repulsive force of the two electrons é W,, =, The total work done by the electrical forces is 27, + ¥,,- The 4ne,(2a) energy required to remove the two electrons is the negative of this, fg ti Ine ,R 4d RB We can check this result in the special case of ¢= R, when the electrons initially sit on. the surface of the sphere. The potential due to the sphere is the same as for a point charge +2e at the center of the sphere. Wis =U-U, 2 2 ee U,=0.U, =2 i eal) i = Ans,QR) AneR 4) Sre,R ‘The work done by the electric forces when the electrons are removed is — Te?/Sis,R and. the energy required to remove them is 7e”/8xz,R. Setting d =R in our general expression yields this same result. 23.53: a) re “il ae) b) The fact that the electric potential energy is less than zero means that it is energetically favourable for the crystal ions to be together. _ 2k? 1)" 23.54: a) U = 2kg? (-4 om ey c) The potential energy is the same for the negative ions—the equations are identical if we examine (a). 2k(1.60 x 10 C)* In(2) _ = 113%10% J. 2.8210 m d) If d=282%10 m, then Y =— ¢) The real energy (-0.80 x 107 J) is about 70% of that calculated above. = 2he? _ — 2(1.60x10" C) _ r 0.535% 10" m b) Ifall the kinetic energy goes into potential energy: 23.55: a) U,= -8.61y 108 J. Qke? U, =U, + K =-8.61 1078 341.02 107 7.5910 J =8.24« 10m" (¢ =5.35x10" m) (Note that we must be careful to keep all digits along the way.) > x- 2.87«10 m 23.56: F, = mg tan@ = (1.50 x10" kg) (9.80 m/s”) tan (30°) = 0.0085 N. Balance forces in xand y sisetios: ) But also: R=iq- Yay. FA _ 0.00858) 0.0500.) _ 45.9 g 8.90x10% € 93.57: a) (i) V= =e /a) ~ In@/b)) = e/a). (i) V= zyme ~ In(b/2))= ne : Gi) ¥=-0. b) Vas =V (@)- Vib) = sen. c) Between the cylinders: a Ys = Fah aes ne OV Van _ Vs 1 a inG/a ar Ol”)? In@/a) d) The potential difference between the wo cylinders is identical to that in part (b) even if the outer cylinder has no charge. 23.58: Using the results of Problem 23.57, we can calculate the potential difference: v1 = => F,, = Eln(oi In@jayr Ofar = V,, = (2.00 x 10* N/C) (In (0.018 m/145 x 10~ m)) 0.012 m = 1157 V. 23.59: a) F = Eq = (1.10 x10" V/m) (1.60% 1079 C)=1.76x 10" N, downward. b) a@=F/m, =(1.76 x10" N)/(9.11x10™ kg) =1.93x10"* m/s’ downward. 0.060 m —< 1 1 2 -< c) tua oy Io =e = 70.9310" m/s (9.23 x10%sy* =8.22x107 m. d) Angle @ = arctan(v, /v,) = arctan(at/y,) e) The distance below center of the screen is: ctan(1.78/6.50) =15.3°. 0.120 m D=d,4+v¢=8.22% 107 m + (1.78 10° 's) ————____ = 0.0411 m. yy ¢ 1) & 50% 10° m/s (a) Use AV, toget d aV=['E-di= dm ya ne a Dey Date 2neyAV In dja a 2m AV In bfa AV dneqr ner rin Bla at outer surface of the wire, r= a= 2222 = SON = 2.65% 10° V/m (amen jy (©) at the inner surface of the cylinder, r =1.00 cm, which gives E=1.68x10' Vim 23.61: a) FromProblem 23.57, _ sy 1 $0,000 V 1 In(b/a)r In (0.140/9.0010") 0.070 m => E=9.72x10' V/m. «10% eS 2 4 _10 2.0010 keV 8029") 3 py otc 9.72108 Vim b) F=Eqg=10mg3¢ 23.62: Recall from Example 23.12 for a line of charge of length a: vn) alla de 4 22] 4a /44x? -a/2 a) For a square with two sets of oppositely charged sides, the potentials cancel and 0. b) Ifall sides have the same charge we have: n3 a’ [44x + af2 a } but here x= 2/2, so: a yada? -af2 pO, | Voted’ +e 0) of 9) Va? + 4s? -@ W2-p}" a a 23.63: a) w-—2 e|-4e rdr {err aR? RB fear vafar-%ej rdr = 2 (gir R Py yej= 2 |e te +e 10, | fee + alx oa Be | ae ale A Tf acca, fl4 a?/x? balan +}(2) 214%, and Ind ta)wat tal sn x) x srenid|(2. (2) --)-f 22 ‘That is, the finite rod acts like a point charge when you are a long way from it. b) From Example 23.12: rt y= 2m yatxita in l4xt/a? +1 ae @4xt—a| 2a yl+x2/a?-1) 2 Uf x<Rik= 25 reR: — Bar fier jaer-2 8 ar : 2 aya 12_101,,f 10 #010" R R22 |, R 2R wz aye 2|3_7 2 LR b) 23.71: a) Problem 23.70 shows that. 2 G-?/R*) for r < Rand V, = 2 torr >R SneyR Azgr = oo Hee sant My = 2 Sxe,R Ane, SxeyR b) If Q>0,Y is higher at the center. If Q<0,V is higher at the surface. 23.72: (a) Points a,b, and are all at the same potential because E = 0 inside the spherical shell of charge on the outer surface. So AV, = AV, = AV, = 0. _kg_@x1? Nm/C’) 50x10" C) AY, R 0.60 m =2.25%10° V. (b) They are all at the same potential (©) Only AV, would change; it would be — 2.25 x 10° V. 23.73: a) The electrical potential energy for a spherical shell with uniform surface charge density and a point charge q outside the shell is the same as if the shell is replaced by a point charge at its center. Since F, =—dU/dr, this means the force the shell exerts on the point charge is the same as if the shell were replaced by a point charge at its center. But by Newton’s 3" law, the force ¢ exerts on the shell is the same as if the shell were a point charge, But q can be replaced by a spherical shell with uniform surface charge and the force is the same, so the force between the shells is the same as if they were both replaced by point charges at their centers. And since the force is the same as for point charges, the electrical potential energy for the pair of spheres is the same as for a pair of point charges. b) The potential for solid insulating spheres with uniform charge density is the same outside of the sphere as for a spherical shell, so the same result holds. ¢) The result doesn’t hold for conducting spheres or shells because when two charged conductors are brought close together, the forces between them causes the charges to redisiribute and the charges are no longer distributed uniformly over the surfaces. 23.74: Maximum speed occurs at “infinity” Energy conservation gives fag 1) 2 1 2 SE = ma Vogt Msn eg MaMa +5 M20 %is0 Momentum conservation: mgvsq = M,spViep ADU Veg = 3V; ep Solve for vj. and v,,,, where r= 0.50 m Vag =12.7 m/s, rj59 = 4.24 m/s Maximum acceleration occurs just after spheres are released. IF = ma gives fade a (910? Nm?/C?) 0% (0.50 m)* 23.75: Using the electric field from Problem 22.37, the potential difference between the conducting sphere and. inutlating sell ia: kQ 1 #Q V=-[E-d?=- [ar =20| --—|>v-=. [pun Pa-ie| eee? 28.16: a) Atr=c:¥=- [a= ur ¢ b) i E.ar-M_o- 1 d) Atr=0:F =kg ye st | since it is inside a metal sphere, and thus at the ¢ a same potential as its surface. 23.77: Using the eleciric field from Problem 22.54, the potential difference between the two faces of the uniformly charged slab is: 4 2 23.78: a) V= He te -16.6V. r 6.5010 b) The volume doubles, so the radius increases by the cube root of bvo R,, =V2R=8.19x107 m and the new charge is Q,., =20=-2.40x10" C. So the new potential is: 12 17, = BQsee = He i OQ ogay Rese 8.19 x10" x >> all, a 2 since in (14a) va. ax 2 yvty st pswany; e222. sincera NEY gs V8 ayy yo y 23.80: Set the alpha particle’s kinetic energy equal to its potential energy: 9 Cy K =U s110 Mev = R282) KAGE) 0.60 x10" C) 7 11.010" eV(1.60x10 eV) =2.15x10 m, Qs, Qs 23.81: V= = 3 AE ee a) R, =O 3 vy) #,--2| = Hp MOAI) _ 310s ap, He Ole Re R/3Y RE Ey 23.82: a) From Problem 22.57 we have the electric field: renig-@oy =p a= 12 r which is the potential of a point charge. ol. eo b) PSR Ba l4 53 3V¥=-|Ear ide r 2 ps ps ope |10" ok ok A late Pot yal, R R R R Rg RR R 23.83: a) B= a v= 5. 80 = RE b) After stecinaiss santa is reached, with charge Q/ now on the original sphere we have: Q, =O) + Quand, =¥, 22.8 -o% R, =QR ok __ 80, ,_ R/RY, RO, QP Dep rae a tReet c) The new potential is the same at cach sphere’s surface: 7-22 mae RR, (+8) @ +R) d) The new electric field is not the same at each sphere’s surface: aA@_ RE RRG+E) RR +R) pak _#O, 10, 2 Re RE d+2) RR +R) 23.84: a) We have V(x, y, 2)— A(x? —3y7 +.2”).So: Ov; OF, Ov k= —2Aad + 6Ayj - 2Ack & ay? & " Hi b) A charge is moved in along the z-axis. So the work done is given by: Q Fl W=q| Bk de=q [(-2d2) de = +(dg)2,° > A= mi 5 Gey 600x107 IT __640-V fen? (15x10 €)(0.250 m) c) E(0, 0, 0.250) =— 2(640 V/m’) (0.250 m)k = -320 V/ mk. 4) In every plane parallel to the x -z plane, y is constant, so: Via, ya) = Ax? + do? Coax? +2? Tee, which is the equation for acircle since & is constantas long as we have constant potential on those planes. @) V=1980 Vyand y = 2m: 22 4 2? = 2OV + 3640 V/")2.00 my 640 V/m" Thus the radius of the circle is 3.74 m. 2 “3 Ge 3.85: a) E,=E, 32 liv l-Hesove | __kGe0 x10) _ ye Oe .2x 10> m\(.67 10" kg) == 7.58%108 mys. b) Fora helium-helium collision, the charges and masses change from (a): 26 x10° m/: 14m 2 2 29 ans)? 7 = MY 67x10 ke X7.58 x10 ny) oy K 2 2 °° 3k 3(1.38 x10" I/K) 99)(1.67 x10" keX7.26 x 10° m/s)? =64%10°K. 3(.38 x10 I/K) * d) These calculations were based on the particles’ average speed. The distribution of speeds ensures that there are always a certain percentage with a speed greater than the average speed, and these particles can undergo the necessary reactions in the sun’s core. 23.86: a) The two daughter nuclei have half the volume of the original uranium nucleus, so their radii are smaller by a factor of the cube root of 2: x10 m r =5.9x10" m. =4.14x101 J 2 2 =I cy? b) y= Hoses _ £(46)" 1.6010" C) 1.17410" m Each daughter has half of the potential energy turn into its kinetic energy when far from each other, so: K=U/2=(4.15 x10" 1/2 =2.07 «107! J. ce) Ifwe have 10.0 kg of uranium, then the number of nuclei is: _ 10.0 kg, "* 36u (1.66 «10 kp/uy And each releases energy U:E =nU =(2.55% 10" \4.15%10" J) =1.0610' J= 253 kilotons of TNT. d) We could call an atomic bomb an “electric” bomb since the electric potential energy provides the kinetic energy of the particles. = 2.55x10" nuclei. 23.87: Angular momentum and energy must be conserved, so: myb = my,% and E, = E, => E, jr + Hite and, =11 MeV =1.76x107? J. Substiouting in for v, we find: . B= 5 Mi (Edn! - agz yn — Eb? = 0, and note g, = 2e and g, =82¢. an @b=10" m= 4 =1.01x10" m Gi)b=10 may, =1.11x 10 mo. Gi) B=10™ m= y= 2.54410" m. 23.88: a) rSa:V= 1- a = p--Bt 6546 Bal @ r>a:V¥=OandE= b) Sa: BA? = % Qa 0 O15 O _ alr Mar? dr pyatnr'dr [ _2r 22 1) E, , 40? + Dvdr) = 2 - aon An(r? +2r dr) @ 3 z & £0 9 a@ aaa = 00)=2/3- c) ry 2a:p(r)=0, so the total charge enclosed will be given by: toe 2 1, nf Q=4x [oor rasan |r? 4 erm see [3° -£] =0. Therefore, by Gauss’s Law, the electric sci must equal zero for any position x > a. An? Ax prgd 23.89: a) F,=mg= ,/d=qE =F q-— : Ves bd) F singel pe bp =Foor- ory 2 3 2pg 3 _4epsd| Pm] 1g, ¢ firme 3 Vy LV 20s Vn Veg 3 3 7 a93 5 «) gaieelOim [aspao Ne/m'y'a0 139.38) HAGA? Ae 916V 2824 ke/m*)(9.80 m/s") 3 a0? n/a _ foa.sixi0 Ne/in yao 1/3939) sory 107m 2(824 ke/m*)(9.80 m/s) 23.90: For an infinitesimal slice of a finite cylinder, we have the potential: av-—_*#@__ Vo-sR? 2h fez 4k n ones av-Hu oe eee (E[t xy +R + (L/2-0) ¥E/24+x) +R? -L/2-x =V= J. the cylinder's axis b) For L< d/2 means C > C/2 and Q> Q/2= 60 pC c) > 2r means 4 > 44,C > 4€, and Q-> 4Q = 480 pC 24.6: (a) 12.0 V since the plates remain charged. ) @ v=$ Q does not change since the plates are disconnected from the battery. p24. a If d is doubled, C 4. 4C,s0 V > 2¥ = 24.0V (i) 4=2r?, soif r — 2r,then 4 + 44, and C — 4€ which means that yo+y=3.0V 4 24.7: Estimate y =1.0cm eur? _ eqx(0.010 m)? Cc 1.00x10 F The separation between the pennies is neatly a factor of 10 smaller than the diameter of a penny, so it is a reasonable approximation to treat them as infinite sheets. c= “450d = = 28mm 24.8: (a) AV = Ed 100V = 10* N/C)¢ d=10* m=1.00cem fod _ egeR? cp ht da d R=4.24x10%m = 424m (b) Q= CY = (SpE)000 V) = 500 pC 24.9: ay © = 2% a_ OL WGy/r) _ 0.180. m)2xe, 1n(5.00/0.50) b) F=Q0/C= 010.0 x10 C)A4.35 x10? F)= 2.30 V =4.35x10"F Ine, ee o = 8 8.77 AH 584. C/E 31.510" F/m > " pay En 2 Hs 60:9) T= es InGs) b) 2-v5-e.sowyarsn10” F/m)=8.19%107! C/m. WAL: a) C/b === __ 2 In /r,) In@.S mm/1.5 mm) b) The charge on each conductor is equal but opposite. Since the inner conductor is at a higher potential it is positively charged, and the magnitude is: _Qnegk¥ _ 276,(2.8 m\(0.35 V) InG,/,) In@.Smnmy/1.5 mm ) =6.56x 10" Fim. O=av =643x10"C. 24.12: a) For two concenizic spherical shells, the capacitance is: ca1| 2 | acy -a, =, 3 = 2 Elyn-y ay, Ea yy = BUG 10" FOIE) _ 9195 kU16x 10 F)-0.150m b) F = 220 V, and Q= C¥ = (116 x10"? F220 V) = 2.55 x10 C. oars: ay CHL ate] 2p 0.148 90.125 mm) ) goa ct Ryan 0.148m—0.125m b) The electric field at a distance of 12.6 cm: 4O sey £(8.94%10"™" FLI20V) gogo yc ror (0.126 m)* c) The eketric field at a distance of 14.7 em: iQ. ACV _ &(8.94x 10" F)120 V) 7 (0.147 my* d) Fora spherical capacitor, the electric field is not constant between the surfaces, E= = 4468 N/C. 24.14: aj 24 Lyd =) _4,_} _ ©, G+, CG (B0+50x10"F) 60x10" F) =, =3.42x10" F. ‘The magnitude of the charge for capacitors in series is equal, while the charge is distributed for capacitors in parallel. Therefore, =O, +Q)=%C,, = (24.0 VIB.A2% 10% F)=8.21 «10% C. Qo 6. G Q,=4Q, =8.21x10% C> Q =3.08 x10" C, and Q, = 5.13107 C. b) Hy = =Q,/C, = B.08« 10? C)X3.00 10 F) = 10.3 V. And = 24.0 V-10.3 V =13.7V. ¢) The potential difference between a andd: V,,= 1 =) =10.3 V. Since C, and C,, are at the same potential, 6,2 9 - 50, c 1 1 1 1 — ge ngs GrD+G 6, 200 +40.) 40) = Cy = 2.40 uP. Then, Oy, + Qs = Q, = Qua = Ca¥ = (2.4010 FY(28.0V) = 6.72% 10°C and 20, - 2,3 QO. = Poss ~ SRO" 2.24 «10° C, and Q, = 4.48 «107 C. But also, Q, =Q) = Q,y = 2.2410" C. b) ¥, =O,/C, = (2.24107 ©)/(4.00« 10% F) =5.60V =F, ¥, =Q,/C, = (4.48 x 10°C)/ (4.00 x10 F) =11.2 V. ¥, -Q./C, = (6.72% 107 C)/(4.00 » 10% F)-16.8 V. c) V,=V,, —¥, =28.0V-16.8V=11.2V. 24.16: a) 1 1,1 1 1 JAA 1 _y, 1 _, GC, G.0x10% F) 60x10" F) = 5.33x10° Fl = C,, =1.88x10" F => Q=VC,, = (52.0 V)1.88x10° F)=9.75 x10" C a .75 x10 C/3.0x10° F=32.5 V. 9.75x10% C/5.0x10% F=19.5 V. ¥,=O01C, WIT: a) QO, = VC, = 62.0 VB.0x10% F)=1.56 x10" C. Q, = VC, = (52.0 V)5.0% 10° F)= 2.610" C. b) For parallel capacitors, the voltage over each is the same, and equals the voltage source: 52.0V. was: C,=(£+4)' = (4 “J! = G25. So the combined capacitance for two capacitors in series is the same as that for a capacitor of area A and separation (d, + d,). 19: C=C, + C, = 2 +20 = 844) | $0 the combined capacitance for bv0 capacitors in parallel is that ofa single capacitor of their combined area (4, + 4,) and common plate separation d. 24.20: a) and b) The equivalent resistance of the combination is 6.0 iP, therefore the total charge on the network is: Q = C,,V.q (6.0 #F\(36 V) = 2.16 x107C. This is also the charge on the 9.0 #F capacitor because it is connected in series with the point b. So: 2 Cy 9.0x10°F Then % =V=%. + =V Wy =36V-M V=12V. = O,= GV, = (3.0 sA)A2 V) = 3.6 «107 C. = Q.= CM, = 11 wE\2 V)=1.32 «107 C. = 0;=On=2-O,- On = 2.16 x10~ C-3.6x10* C-1.32x107 C. =4.8x10° C. So now the final voltages can be calculated: _ OQ, 483x107 C “CG, 60x10" F y, Qa 48x10" € 12 [5A e Rn Cy, 12%10°F c) Since the 3 #F, 11 »F and 6 capacitors are connected in parallel and are in series with the 9 F capacitor, their charges must add up to that of the 9 F capacitor. Similarly, the charge on the 3 pF,11 pF and 12 F capacitors must add up to the same as that of the 9 x capacitor, which is the same as the whole network. In short, charge is conserved for the whole system. It gets redistributed for capacitors in parallel and it is equal for capacitors in series. %, =8V. 24,21: Capacitances in parallel simply add, so: 1 1 _——— | =) pP x =57 aR. (144040 vom)" Ha) PEER BE SS SE 24,22: a) C, and C, are in parallel and so have the same potential across them: 4% ~&_ AD O10TS =1333V C, 3.00x10° F ‘Thus Q, =C, = (13.33 V3.0 x 10 F)=80.0x10% C. Since Q, is in series with the parallel combination of C, and C;, its charge must be equal to their combined charge: 40.0% 10°C + 80.010" C=120.0% 10°C. b) The total capacitance is found from: 1,1 1 1 op GG 900x10°F 5.00x10° F Cy = 3.21 and. 8 7, = Ga OIE _ gy gy Cy 3.210 F 24.23: V = QC, = (150 C9 /(3.00 pF) = 50 V C; and C, are in parallel, so V, = 50V ¥,=1200V-4=70V 24.24: a) V = O/C = (2.55 pC) /(920% 10? F) = 2772 V. b) Since the charge is kept constant while the separation doubles, that means that the capacitance halves and the voltage doubles to 5544 V. ce) U=4CV? =4(920x 10"? F\(2772 Vy? = 3.53 x10” J. Now if the separation is doubled, the capacitance halves, and the energy stored doubles. So the amount of work done to move the plates equals the difference in energy stored in the capacitor, which is 3.53x107 J. 24.25: E=V/d = (400V)/(0.005 m)=8.00x10* V/m. And u=42,E? =4.,(8.00x 10* V/m)’ = 0.0283 J/m*. 24.26: a) C= Q/F = (0.0180 C)/(200 V) =9.00 x10" F an by Cate gid _ 0.00210 0.0015 m) _ 9 9159 d & & ©) Enag = Vang | d=? Vag = Equcd = (3.00 x 10° V/m)(0.0015 m) = 4500 V. _@' _ 080x107 Cf === a1 80x10. 2C 2(9.00 x10 F) au 24.27: U = ECV? = 4(4.50x107 F295 V)* =19.6J. 24.8: a) Q=CY,. b) They must have equal potential difference, and their combined charge must add up to the original charge. Therefore: mae Beant aso 2, - 2, =O= 0%, G=Cand c, = 50 2 = _& 2 € C/d 3 2 osov-2=22-2 39-50 30,-70s0v-2-72_2y, 3 a v-{2.2)) se" ‘i 240") 12 Lex 2Aqg G4} 2 C 3C 3 d) The original Uwas U =4CV,' = AU =2C¥,’. e) Thermal energy of capacitor, wires, etc., and electromagnetic radiation. 2 oe 24.29: a) Up fen ae 0 b) Inctease the separation by die => U = ©4880" (+ a/.x). The change is then 25d. ¢) The work done in increasing the separation is given by: aW -U-U, 2x9" Fax o =-U-U,= 3 3 F= = 2A 2A d) The reason for the difference is that £ is the field due to both plates. The force is QE if Fis the field due to one plate is Q is the charge on the other plate. 24.30: a) If the separation distance is halved while the charge is kept fixed, then the capacitance increases and the stored energy, which was 8.38 J, decteases since U = Q" /2C. Therefore the new energy is 4.19 J. b) Lt the voliage 1s Kept ted while the separation 1s decreased by one halt, then the doubling of the capacitance leads to a doubling of the stored energy to 16.76 J, using U=c¥? /2, when V is held constant throughout W311 a) U= 0? 2c Q= J2UC = 2(25.0 15.00 x10 F) = 5,00x 10 C The number of elecirons JV that must be removed from one plate and added to the other is N=Q /e=(5.00 x10% C)/0.602 x10" C) = 3.12 10! electrons. b) To double U while keeping Q constant, decrease C by a factor of 2. C= e,4/d; halve the plate area or double the plate separation. 32s CH= 2 a8 200 3 a7 10 farad v 240V Since C = Ke, d/d for a parallel plate capacitor Key _ (1.00)(8.85 x10? C? /N- m?)(2.60 x10 mm?) Cc 3.417% 107? farad =6.734x 107m d= The energy density is thus ACP 4GA210 fad 40 Vy? ” = Ad (2.60%10%m)(6.734 10m) = 563x107, mi = 160x107 C 2 _ 23.2010? J) v 4.00V 24.33: a) u=l0V > 9= b) C__ 2xey cn L ners) % = exp(2ne, L/C) = exp(2xe, LV /Q) => = exp(2ne, (15.0 m)(4.00 V)/(1.60 x10 C)) = 8.05. ots 24.34: a) For a spherical capacitor: Cub _ 10.100 my(0.115 m) ky—7, (115 m—0.100 m) =>V =O/C =(3.30x 10° C)/(8.53x 10"! F)= 38.7 V. au A ») udev? - 59x10 BGETY) =8.53x10"F = 6.38 x10° J. (ka) a (EC) _ ok? Q20V¥' 94x10" BY [ 2) a 2 (0.126 my" 24.35: a) w= 228? =| 2 Y r 2 > w=1.64«107 J/m’. b) The same calculation for r= 14.7 cm > w= 8.83 x10 J/m>. c) No, the eleciric energy density is NOT constant within the spheres. 1 8.00x107 ©)? 322%, (0.120m)* b) If the charge was -8.00 nC, the eleciric field energy would remain the same since U only depends on the square of E. =1.11x104 J/m'. 14136: a) we Lge? al g| @ 2 2° dns, r? 24.37: Let the applied voltage be V. Let each capacitor have capacitance C. U=4C¥? for a single capacitor with voltage ¥. a) series Voltage across each capacitor is 7/2. The total energy stored is U,= {2c /2)") =4cv? parallel Voltage across each capacitor is V. The total energy stored is ohev?)=cv? 4U, b) Q=C¥ for asingle capacitor with voltage’. Q,=ACV/2)=CV; Q,=ACV)=2CV; Q=20, c) E=V/d fora capacitor with voltage ¥ E,=V/2d; E,=V/d; E,=2E, 24.38: a) C = Key A/d gives us the area of the plates: _ Cd (5.00 x10" farad)(1.50x 107 m) “Key (0008.85 10 ’C?/N- m2) Wealso have C= Ke, d/d = Q/V,300 = Ke, AV |). V /d ia the electric field between the plates, which is not to exceed 3.00 x10 N/C. Thus Q= (1.00)(8.85x 10 C?/N-m*)@.475 x 10% m*)(3.00x 10" N/C) =2.25x10C b) Again, O= Ke,A(V /d)=2.702,A(V /@). If we continue to think of V /d as the electric field, only X has changed from part (a); thus Q in this case is QO 1N)E 25x10) = 6 OR x10 =8.475x107 m? 24.39: a) 0, = ey (3.20 — 2.50) x10° V/m) = 6.20107 C/m?. The field induced in the dielectric creates the bound charges on its surface: b) =F 20x 10° V/m E 2.50x10 V/m =1.28. 24.40: a) Ey = KE = (3.60)(1.20x 10° V/m) = 4.32108 V/m=> o= 2,2) = 3.82x10° C/m’*. b) o,=9| (3.82% 10% C/m?j0 — 1/3.60) = 2.76 x 107° C/m?. Ue ©) U=4CV? =udd =1 Ke BE’ Ad =U =1(3.60)e, (1.20% 10° V /mn)?(0.0018 m)\(2.5 104 m?)=1.03% 107 J. Wl: C= = 0.0135 m’. Kid _KeAE CV __(1.25x10" FY(S500V) d v Ke,E (.60}e,(1.60x10" V/m) 24.42: Placing a dielectric between the plates just results in the replacement of ¢ for s, in the derivation of Equation (24.20). One can follow exactly the procedure as shown for Equation (24.11) YAB: a) 6 = Key = (2.6)e) =2.3 x10! C?/ Nim*, D) Fan = End = (2.0%10" V/m)(2.0x 10° m)=4.0% 10" V. c) E= km? o =6E =(2.3x107 C?/Nm)Q.0x10" V/m)= 0.46 x107 C/m’. £9 0.46 «107 C/m?)d -1/2.6) = 2.8% 107 C/m’?. dd: a) AO=O-O, = (K-10, =(K- NGM, = 2.12.5 x10" FU2V)= 6.3x10° C. b) 9 =O0-+)=(9.3x10% CM-1/3.1)=6.3x 10°C c) The addition of the mylar doesn’t affect the electric field since the induced charge cancels the additional charge drawn to the plates. 14.45: a) U,=Lovt ay = [242 = [085107 D gry. 2 C, ¥G.60x10" F) U__ (232x109 41.85%10 TF) _ 227) CV? (360x107 FYI0.1 Vy" b) =i Key" =>K= 24.46: a) The capacitance changes by a factor of K when the dielectric is inserted. Since Vis unchanged (The battery is still connected), Later Qater _ 45-0PC _ ¢ 1 99 Come Qnetre 25:0PC b) The area of the plates is ar? = (0.0300 m)? = 2.827 x10“ m’, and the separation between them is thus _ Key (1.00)8.85 107? C? / N-m?)(2.827 x 107 m?) d eu Cc 12.5%10 farad = 2.002 x10" m Before the dielectric is inserted, c= Kad 2 ad Vv p-@4- 25.0% 10 C\2.00* 107 m) KeyA (1.00)(8.85 x1 07? €?/N -m?\(2.827 x 107 m’) =2.000V ‘The battery remains connected, so the potential difference is unchanged after the dielectric is inserted. c) Before the dielectric is inserted, eo | 25.010? C KA (885x107 C?/N-m*)1.00)2.827 x10 m’) = 999 N/C Again, since the voltage is unchanged after the dielectric is inserted, the electric field is also unchanged. W.AT: a) before: V, =Q, /C, = (9.00 x10 C)/(3.00x 10 F) = 3.00 V after: C = KC, =15.0F;0=Q, V = Q/C=0.600 V; V decreases by a factor of K b) E=V/d, the same atall points between the plates (as long as far from the edges of the plates) before: E = (3.00 V)/ (2.0010 m) =1500V/m after: E = (0,600 V)/ (2.00 x10 m) = 300 V/m 14.48: 14.49: 24.50: ree a) KE- => KEdnd? =-4 > g=—4 £9 & Aned = 4g + b) fé-aa Bor IF +9. yn gt 44% , pp At % re % 4a0,d" = Gout = 9 + 9 = 9/ K c) The total bound change is g, = ¢(+—1) xs a) Equation (25.22): KE. ad Se > KEA=2> E= b) V=ni= 24 OF oo. Kid oe) CaL aL ft xc, Food d 2 =4.8x10" F b) Q=C¥ =(48x10'F) 02 V)=0.58x 10° ¢ c) E=V/d =(12 Vy(4.7% 10% m)=2553 V/m. d) U=4CV? =4(4.8 x10 FYI2 Vy = 3.46% 107 J. e) If the battery is disconnected, so the charge remains constant, and the plates are pulled further apart to 0.0094 m, then the calculations above can be carried out just as before, and we find: a) C=241x10"F b) Q=0.58x107 CG @ _ 0.58x10" CY = a6 91x 0Pd 2C -2(2.41x10" FY c) £=2553V/m sd) Ua 24.51: Tf the plates are pulled out as in Problem 24.50 the battery is connected, ensuring that the voltage remains constant. This time we find: a) C=24x10"F b) Q=2.9x10"C c) E= FAY 4 acter & m 2 au nt (24x10 FO2VY* aay ig? b) Ak rearranging, the E elds should be calculated. Use superposition recalling E= £7 fora single plate (not ~£; since charge Q is only on one face). 2 2), (2)_e Desk ar) (iS ) Gel G&) wa between 3 and 2: E= (2 224 ( 2) + 2.) + 2. 2 Dek?) \ Qepd Deol? J, (Qe? J, sob! = —o_ Q Q bev zant be I: iy le) Gl) between | and 3: E= 2 2 ss =(hae")2e = +2 \ze 204 2 x RE) eel 2a Ot a oct, vt et awe This is the work required to rearrange the plates. 24.53: a) The power output is 600 W, and 95% of the original energy is converted. => B= Pt= (2.70% 10°W) (1.48 %10° s) = 400 J +, E, = 82 = 421. 2U _ 4217) b) uater sc == 21) _ oos4R 2 ve a2svy (4.20107 me, “d7.00%10% m = C=C, +0.25 pF=7.81x10" F Bur C= 80 a g’ — Heo 420x107 me _ 4 76, 10-4 a’ C 781x10°F Therefore the key must be depressed by a distance of: 7.00 107 m- 4.76 «107 m= 0.224 mm. 2.54: C, =531x104F WiSSE a) aRty, OS Ea b) At the scale of part (a) the cylinders appear to be flat, and so the capacitance should appear like that of flat plates. 24.56: Originally: Q, = CV, = (9.0 pF) (28 V) = 2.52107 C; Q, = C,¥, = (4.0 uF)» (28 V)=11210% Cand C,, = C, + C, = 13.0 So the original energy stored is U =4C,¥? = 403.0% 10 F) (28 V)* = 5.10 x10" J. Disconnect and flip the capacitors, so now the total charge is Q = Q, — Q, = 1.4107 C, and the equivalent capacitance is still the same, C,, =13.0 #F. So the new energy stored is: _@ _ d4xio* cy 2, 203.0x10° Fy = AU =7.45 x10 J-5.10 x10 J=- 4.35107 J. =7.54.%10% J 2457: a) C,, = 4.00 uF’ + 6.00pF = 10.00 pF, and Q,.,, = C,, ¥ = (10.00 iF) (660 V) = 6.6x 10°C. The voltage over each is 660 V since they are in parallel. So: Q,=CV, = 4.00 pF) (660 V) = 2.64107. Q, = CV; = (6.00 pF) (660 V) = 3.96% 10° C. D) Qe = 3.96% 10 C—2.64 10° C= 1.32 x10" C, and still C,, = 10.00 F, so the voltage is V= Q/C= (1.32107 C)/(10.00 uF) = 132'V, and the new charges: OQ, = GF, = (4.00 pF )132 V) = 5.28% 10 C. Q) = CF, = (6.00 pF\(I32 V) = 7.92107 C. 24,58: a) egy C, = $+ £=C. So the total capacitance is the same as each individual capacitor, and the voltage is spilt over each so that V = 480 V. Another solution is two capacitors in parallel that are in series with ovo others in parallel. b) If one capacitor is a moderately good conductor, then it can be treated as a “short” and thus removed from the circuit, and one capacitor will have greater than 600 V over it, 2459: a) at A Ca GA the 1.°2,2 055 C, =C, =C, 90 —=— 4+ =-6, > ¢, CC 3G, b) Q=C¥ =(2.52 uF\(220 V) = $.54x10% C=0, = 9, SV, =V, = (5.54107 C)/ (8.4% 10° F)= 66 V. SoV, = 220- 2(66) = 88 V > Q, = (88 V4.2 pF) = 3.70107 C. Also = 7, = 488 V) =44.V 3 Q, =O, = (44 V4.2 pE)=1.85x107C. 24.603 a) With the switch open: C,, = (Gly + aig)" + (oly + ats) ')= 4.00 oF > Opa = CV = (4.00 2B) (210 V)=8.4 x 10 C. By symmetry, each capacitor carries 4.20 10“ C. The voltages are then just calculated via V=O/C. $0: Vig — QUC, = 140 V, and V, = OIC, = 70 V 3 Vy Vig — My = T0V. b) When the switch is closed, the points c and d must be at the same potential, so the equivalent capacitance is: f4 1 y C= + 4] 245 pp “G1 OmF | G10 mF > Opus = Ca’ = (4.50 2B) (210 V)=9.5 x10 C,and each capacitor has the same potential difference of 105 V (again, by symmeiry) c) The only way for the sum of the positive charge on one plate of C, and the negative charge on one plate of C; to change is for charge to flow through the switch. ‘That is, the quantity of charge that flows through the switch is equal to the charge in Q, - Q, = 0. With the switch open, Q, =Q, and Q, — Q, =0. After the switch is closed, O, - O, =315 nC; 315 uC of charge flowed through the switch. =i 1,1 , 1) loa 84ApE 8ApE 4.2 pF => Q=C,V = 2.12 F) G6 V)= 750x107. b) U=4CP? =4(2.1 pF) (36 Vy’ =136x10° 1. c) If the capacitors are all in parallel, then: Cu = (8.4 HE +84 pF + 4.2 uP) =21 xF and Q = 3(7.56 x10°C) =2.27 x10 C, and V = Q/C =(2.27%10* C)/(21 nF) =10.8 V. d) U=4CV? =4(21 pF) (10.8 Vy’ =1.22 10° J. 24.61: a) car 24.623 a) C, ——s (4.0pF 6.0 pF 3 0=CV = (4x10 F) 600 V)=1.58x 10°C and V, = Q/C, = (1.58%10% C)/(4.0 pF) = 395 V = ¥, = 660'V — 395 V = 265 V. =i } =2.4x10°F b) Disconnecting them from the voltage source and reconnecting them to themselves we must have equal potential difference, and the sum of their charges must be the sum of the original charges: Q.=GV and Q, =CV = 20=0,4+0,=(G4+6,)¥ -3 29 _ 58x10" C) _gigy G+, 10.0x10°F = Q, = (4.00x 10° F316 V) =1.26 x10" C. = Q, = (6.0010 F316 V)=1.90x107 C. >= 24.63: a) Reducing the furthest right leg yields C= (gle + aoe + make)" = 2.3 xF=C,/3. Itcombines in parallel with aC, > C= 4.6 pF + 2.3 pF=6.9 pF=C,. So the next reduction is the same as the first: C = 2.3 »F = C,/3.And the next is the same as the second, leaving 3C,’s in series soC,, = 2.3 nF = C,/3. b) For the three capacitors nearest points a and b: Qo, = Cag¥ = (2.3% 10% FY(420 V)=9.7 «107 C and Qo, =C,V, = (4.610 F)(420 V)/3 = 6.44 x107 C. c) V4 =4( V)=46.7 V, since by symmetry the total voltage drop over the equivalent capacitance of the part of the circuit from the junctions between a,c and d,b is V, and the equivalent capacitance is that of three equal capacitors C, in series. V.. is the voltage over just one of those capacitors, ie., 1/3of 2 V. VW.64: (8) Coa, =C, + Cy + Cy = 60 LE Q= CV = (60 pF) (120 V) = 7200 xe 1 1 1 ajay GGG Capiy = SAS HE Q=CV = (5.45 nF)120 V) = 654 nC 24.65: a) Qiis consiant. with the dielectric: V = Q/C = O/(KC,) without the dielectric: ¥, = O/C, ¥,/¥ =K,s0 K= (45.0 VY(1.5V)=3.91 b) AB Als 2/3 2a/3) Let C, = 4 4/d be the capacitance with only air between the plates. With the dielectric filling one-third of the space between the plates, the capacitor is equivalent to C; and C; in parallel, where C, has 4, = 4/3and C, has 4, = 24/3 G= EG/3,C, =20,/35C, =C, + =(C,/3)(K +2 Qg_@/ 3 \_ 3 je wi) ar 2s) wsov(=5) BEN oa 24.66: a) This situation is analagous to having two capacitors C, in series, each with separation 4(d — a), Therefore C = & + ay =4C,=tgta= d d-a dd-a °d-a ¢) As a30,C 3G. Andasa3d,C 30. Sod aa" 24.67: a) One can think of “infinity” as a giant conductor with 7 = 0. b) C=2= gig = 40e,R, where we've chosen V = 0 at infinity. ©) Cony = 46 (Reg y =46)(6.4%10° m)=7.1x10~ F. Larger than, but ‘earth comparable to the capacitance of a typical capacitor in a circuit. 2.68: a) r Riu=beE? => 2 @ | 3 Amey?) 32x eqr" 2 ye 2 °) U = fuav =n [rude 2 fe g g Sey g 12 2 Tae d) This energy is equal to which is just the energy required to assemble all the charge into a spherical distribution. (Note, being aware of double counting gives the factor of 1/2 in front of the familiar potential energy formula for a charge Qa distance R from another charge Q.) e) From Equation (24.9): U/ = Problem (24.67). 28 Tao from part (¢:) > C =42,R, asin 2 ye 2 W609: a) rR: wage = 2(8) war or Ris [ud = 4 fred =f 1 wy HO. 4 ry 3R 2 1 » 24.70: =s@E = = a) Bape ta ( 2] Brey 2 dp b) U= Jud =2nb furar = Bay U Jr). 4neyt LS Ane c) Using Equation (24.9): y-2.- tes !7,)= 22 tnt r= U of pat (). 2C° Ameye ** ° 4 aiset 1 WT: C= (4) +(24) wel (2 wale ai2 a2 eA K, 3c -294{_KK ) ad (K+, 24.72: This situation is analagous to having two capacitors in parallel, each with an aread. So: Ald é- a Af2_ end d C2616 ay Kit Ki). > Cm? 273: a) B= —2- = SSO Cin" 107 Vim. Key CAe, b) V= Ed =(1.0x10" Vim) (5.0x 107 m)= 0.052 V. The ouside is at the higher potential. ¢) volame =107 m? = Rs 2.88%10% m => shell volume = 4R"d = 42(2.88x 10% m)’(5.0x 10° m)=5.2x10" m’ SU =ul = (LKegE*)V =4(5 Ae, (1.0% 107 Vim)? (5.2% 10? m?)=1.36 x10 J. os pay. Kég yy _ (2-50)¢9(0.200 m*) (3000 V) _ 9 Wd: a) Q=CV = SEE y = OE 1.33 x10 C. b) Q,=Q(-1/K)= (1.3310 C) (-1/2.50) = 7.98% 107 C. 6 dj pea 2 ee Lie, é Kad (2.50)e, (0.200 m’) d U=LOV =20.33010 C) (3000 V)= 2.00107 J. U 2.0010" J e) w= _e Ad (0.200 m?) (0.0100 m) u=4Ke,E” =4(2.50)s, (3.01% 10° Vn)’ = 1.00 Jim’. £) In this case, one does work by pushing the slab into the capacitor since the constant potential requires more charges to be brought onto the plates. When the charge is kept constant, the field pulls the dielectric into the gap, with the field (or charges) doing the work. .00 Jim? => or 24.75: a) We are to show the transformation from one circuit to the other: Gireuil 2 4 = 27% here g,is derived from V4: 3q-—2E _|4_%|axfa_-% OCC eC, Cc C. From Circuit 2: on Se Being E12 log Land QC Gp G Ge UTE: 1 11 Via ab + ta +a) —4+>} Q G Gy Cc, CG Setting the coefficients of the charges equal to cach other in matching potential equations from the two circuits results in three independent equations relating the two sets of capacitances. The set of equations are: 11 1o1)11 11 1 1 z-4{ "KG, -z)he-4 "KG, KC, C3; KC,C," From these, subbing in the expression for K,, we get: = C6, +6,6,4+CC)/C,. =C6,+6,0,+C0)/C,. O=€C,466,4+C0)/C. b) Using the transformation of part (a) we have: ra 6 ‘ 7 Ye 3¢ MPN fee Where C, =126 pF, C, = 28 uF, C, = 42 pl, C, = 42 B,C, =147 pif, and C, =32 2F. Now the total equivalent capacitance is: “1 c,-| e+e 4] -s0 oe, 72 )F 126pB 348 ,F A7)B 72,8 where the 34.8 «F comes from: 2b 2 34.8 xP Lecce isl wcihee elena | 42 pE 32 nF 28 uF 42 bP c) The circuit diagram can be re-drawn as shown on the next page. The overall charge is given by: O=C.¥ = (14.0 pF) 36V) > O=5.04x107 ‘And this is also the charge over the 72 iF capacitors. Next we will find the voltage over the numbered. capacitors, and their associated voltages. Then those voltages will be changed back into voltage of the original capacitors, and then their charges. Oe, = Oo, = On = 5.04% 107 C 5.04107 € = a2 a aan > 147 x10 F = Veo, = Veo, = G6.0— 7.00 — 7.00 - 4.00 - 3.43) V = 14.6 V. “ A But C,(C,C,) = (é-2) =168 AF and C,(C,C,)= (4 +4) =18.2 4H, 30: CC, Cs Qo, = Qe, = Ve,e,Caute,c,) = 24510" C. Qo, = Oo, =¥o.0,Coatesoy = 2-4 X10 C. 2%, - 2% _gay, Vo, - 2 63M = oo. =58V, Vo, = oe =8.3V. C, Gq q & 2, = Vo, + ¥o, = Vig =13 V > Qyg = hy = 2.3107 © Vg = Vo, + Vo, = Voy =10V = Qy, = CyVoy = 2.8107 C. Vg =Vo, +Vo, = Vis =9-V 2 Ops = CosVig =2.6 «10 C. Tg =Vo, +Vo, = Voy =12 VQ, = Cy, = 25x10" C. Vy, =Vo, Vo, =Vg=25 V2 O,= GN, =15x10°C. 24.76: a) The force between the two parallel plates is: Pook “(Yad O° eg Qed eg A 22" b) When ¥ =0, the separation is just z,. So: 2 Fusing = 4K — pelt = 223-2272, a 0 c) For 4= 0.300 m?, z, =1.2%107 m.k = 25 Nim,and ¥ = 120 V, 224 — (2.4%107 m)z? + 3.82% 107" m= 06> z = 0.537 mm, 1.014 mm. d) Stable equilibrium occurs if a slight displacement from equilibrium yields a force back toward the equilibrium point. If one evaluates the forces at small displacements from the equilibrium positions above, the 1.014 mm separation is seen to be stable, but not the 0.537 mm separation. & 77: a) C, -E@-yL+ xk1)= SL +(K -)x). b) ay =dacw" where C=C, + ee dx +dxK) ~K=DaP"L 5. 2D =AaUu= se (K- . c) Ifthe charge is kept constant on the plates, then: dl, Dee Decal © (£+(K-1 dd UF = —C¥* =—C¥*4) — = EH(K 1a) and U5 CF? = GPE a =aUs at —Ndx |= AU=U-U, (Koleh' Ey 2 2D d) Since dU =—Fadx = “Ns”! ay. then the force is in the opposite direction to the motion. dx, meaning that the slab feels a force pushing it out. 24.78: a) For a normal spherical capacitor: C, = Azcsy Here we have, in effect, two parallel capacitors, C, andC,,. %. = KOs _ yaks, | 72% Gon 2 non) 2 by Using a hemispherical Gaussian surface for each respective hal 2 ort =o ge 2. > and £4 = 55, oe 2 Key WKeyr® 2 & Qnegr But Q, =¥C, and Q, =¥C,,0, +0, =0 PO ko So: =K (+K)= =—<—and Q, = : 0: O, = QO, => Q,1+K)=O> 9, = 7% O..-ee 259 = #5 1 2 @ _@ 1 _2 @ 2 2 and E,, = 2 2 14K 2aKeyr? 14K Arey” 14K Ker 14K AnKe,r c) The free charge density on upper and lower femighreane _ 2g g (de = 4m (1+ K) and (25, Je = ia? “dere KY xb OE asses TO _ 6 De sos RO __ (5)e= 4 4B a Paes By (K-) Q K _K-1@ Ko dar K+1 K+ 4er" (K-) Q@ K K-19 =e, (= = 15 =A, AWE dor K+1 K-41 4m, ¢) There is zero bound charge on the flat surface of the dielectric-air interface, or else that would imply a circumferential electric field, or that the electric field changed as we went around the sphere. d) 9, = 0, 0-1/K)= 24.79: a) 2 4). 2¢4.2)60(0.120 my" 9 ag 19? F, by c= 4 = d 4.5107 m 24.80: a) The capacitors are in parallel so: _ gL _sW(L-h) , KeyWh _6WL(,, Kh), d @ d d e Ed, =(144_*)} EE b) For gasoline, with K 1 z = full: Kyy| b= = 4 2 4 a Saux (n=4 Jean. 4 4 ©) For methanol, with K = 33: 1 L 1 L = full: Ky | h=—]=9; — ful: Ky| k= = 4 a 4) “2 a: 3) -. SL” —full: K,,| k= — |= 25. 4 «| “) d) This kind of fuel tank sensor will work best for methanol since it has the greater range of Ky values. WA: Q= = (3.6 A)3)3600 s) =3.89 x10" C, Loh: 1 1.24; > fall: Ka(t= 25.2: a) Current is given by Z b) F=ngyA 2 = Oe $8.75x107 A. I 8.75%107 A NES = 28 19 3 2: agd (5.8 %10")0.6 x10? CKnd.3x10° m)) =1.78 10% m/s. 485A 25.3: a) y, af BA ™ 4 'ngd B.5x10")1.6 x10 Ch(m/4 (2.05 x10 m)*} =1.08 «10% m/s => travel time = ae aa” 85749=110 min b) If the diameter is now 4.12 mm, the time can be calculated using the formula above or comparing the ratio of the areas, and yields a time of 26542 s =442 min. ©) The drift velocity depends on the diameter of the wire as an inverse square relationship. 25.4: The cross-sectional area of the wire is Asap? = x(2.06x107 my =1.333 x10" m’. ‘The current density is 6.00 %10° A/m? 4 Wehavev, =J/ne ; Therefore J 6.00 10° A/m? nee = 6.9410 Sleetrons ve (5.40107 m/s)(1.60x 10 C/eleciron) m 25.5: J = nlg|y,, 0 J/y, is constant, Alva ~Filvea> Yq = Vn Sy /T,) = Vy (L,/L) = 0.20 x 107 m/s)(6.00/1.20) = 6.00 x 10 m/s 25.6: The atomic weight of copper is 63.55 g/mole,and its density is 8.96g/cm’. The number of copper atoms in 1.00 m° is thus (8.96 g/em® )(1.00x 10° cm‘/m*)(6.023 x 10" atoms/mole) 63.55 g/mole =8.49 x10" atoms/m°* Since there are the same number of free electrons/m? as there are atoms of copper/m* (see Ex. 25.1), The number of free electrons per copper atom is one. 25.7: Consider 1 m? of silver. density =10.5 x10° kg/m’ , so m =10.5 «10° kg M =107.868 x10™ kg/mol, 0 2 = m/M = 9.734 «10° mol and N=nN, = 5.86%10™ atoms/m’* If there is one free electron per m*, there are 5.8610" free electrons/m?. This agrees with the value given in Exercise 25.2. 25.82 8) Qniy = (iq + My Je = (3.92 X10 4 2.68 x 10° 1.60 x10" C)= 0.0106 C = Loar _ 10106 C _ 9 106 =10.6 mA t 100s =p b) Current flows, by convention, in the direction of positive charge. Thus, current flows with Na* toward the negative electrode. # 5 % ° 0.65 25.9: a) Q=| I at=[(S5- 0.651 at = $54) += 329. a a b) The same charge would flow in 10 seconds if there was a constant current of: I=Q/t=(290)/@s)=41.1A. 28.10: a) J == ayheae — 681 x 10° Alm’. b) E=pJ-=(1.72%10" Q- my6.81x 10° Aim?) = 0.012 V/m. c) Time to travel the wire’s length: _ 1 _Ingd _ (4.0 m\@.5x10" /m'* 1.6 x10? C)2.3x107 m)? “vy LO 3.6A = 1333 min » 22 hrs! =8.0x10's 8 rst: R= LE A-RXIW Q MIC40 mM) _ 9 950, 4 (7/4)Q.05 x 107 my? =9.75 m. Pp 12x10 Qm 95.12: R= 51 = A 25.13: a) tungsten: pepe © 25x10 O/m*)(0.820 A) _ A (#/4)(3.26 x10 my? b) aluminum: Ee pe _ pl _ (2.75 *10* Of m? 0.820 A) A (#/4)G.26% 1075 mj? 5.1610 V/m. = 2.70107 V/m. xd, _ rade? ly dey AP Aw 25.15: Find the volume of one of the wires: Raed 59 dae and A R _1.72 x10" Ohm-m)(3.50m)* 0.1250hm 2 volume Az = = = 1.68610 meb m= (density)¥ = (8.9x10° kg/m" 1.686 x10 m*)=15 ¢ 25.16: 2 PSM 1.625 eam t R=ps 2=2an (per coil) » 75 coils A=m RA_ Ra Rr “T (Qm)7S 150%, (1.74 Q)d.625 x 107 my? ~~~ 150(.75%10™ m) =1.75x10° Q-m 25.17: a) From Example 25.1, an 18-gauge wire has 4 =8.17x107 cm? I= JA= (0.010 Alem? (8.17 x 10% cm?) = 820 A b) 4=J/J = (1000 A)/(1.0 «10° Ajom?) = 1.0107 em? A=ar sor=A/x =4.0x10" cm’/x=0.0178em d@=2r=0.36 mm 25.18; Assuming linea variativu uf Wie vesistivily with teuuperauue. p=poll+ a —-T)] = poll + (4.5% 107/°C)(320 - 20)°C] =2.35p5 Since p= E/J, the electric field required to maintain a given current density is proportional to the resistivity. Thus E = (2.35)(0.0560 V/m) = 0.132 V/m ee 15.9: R=2E HE PBT ss 197! A FL 1.80m 25.20: The ratio of the current at 20°C to that at the higher temperature is (0.860 A)/(0.220 A) =3.909.Since the current density for a given field is inversely proportional to p(p = E/J), The resistivity must be a factor of 3.909 higher at the higher temperature. Lal+a-h) Po 2-1 = T=7,4+2—=20 gp 80M = 666°C a 4.5 x 104/°C ve oh ph, ia (6.00 A\(2.75 x10 Q- m)(1.20 m) 25.2: E (1.50 V) = 2.05% 10 m. +t a5.am p= RA _VA_ SOV R654 107 mY ay gr oe Loi (17.6 A)(2.50 m) 2.28: 9) Pa thn EA 2 049 Vine 4(084 10" mY yy p (2.44 «10 Q-m) >) Pom 2b _GUIAI24 x10" O-mVE4m) 9 454, A (/4)(0.84 x 107 my? °) Re 2 pee i 111A 25.24: Because the density does not change, volume stays the same, 30 Ld = (2£)(4/2) and the area is halved. So the resistance becomes: Ra LOE) _ Ph _ gp. 42. A That is, four times the original resistance. RAJ_ REV _0938V a er a ar Te RAV 0.938 ¥ 0) 8-5-5 Gama Alm WOTS L JE (4.4010 A/m?)(0.75 m) =1.25 V/m. =2.84x10° O-m. 1.512 2-148492 =e 1.35107 CF (B4.0°C— 20.0°C)0.484 2) b) Rp -R =Ral, -T)>R, 0.0158 2. _ 215.8 Q- 217.30 = CHIT SG (-0.0005° C)(217.3.Q) 25.29: a) If 120 strands of wire are placed side by side, we are effectively increasing the area of the current carrier by 120. So the resistance is smaller by that factor: R=5.60x10* Q/120=4.67 x10" 2 b) If 120 strands of wire are placed end to end, we are effectively increasing the length of the wire by 120, and so R = (5.60 10 Q)120 =6.72 x10 25.30: With the 4.0 load, where r= internal resistance 126 V=(r+ 4.00) Change in terminal voltage: AV, = rf =12.6V-104.V=2.2V 22V + Ss r - Substitute forf: 12.6V =(r+ soa) * ~) Solve for r: r=0.8462 1.72 » 10 Qm)(1 00 » 10°m) x(0.050m)* V = IR = (125A)(0.2190) = 274V b) P=VE=Q74 V)(125 A) = 3422 W = 3422 Ws Energy = Pt = (3422 J/s)(3600 s) = 1.2310 J 25.31:a) R= & = 0.2192 28.32: a) V, -€—V, -24.0V —21.2V=28 Vs r-28 V/4.00A = 0.7002. b) Vy =21.2V = R= 21.2 V/4.00 A = 5.302 25.33: a) An ideal voltmeter has infinite resistance, so there would be NO current through the 2.0 @ resistor. b) My resistance. c) The voltmeter reading is therefore 5.0 V since with no current flowing, it measures the terminal voltage of the battery. =4.0'V; since there is no current there is no voltage lost over the internal 25.34: a) A voltmeter placed over the battery terminals reads the emf: € = 24.0 V. b) There is no current flowing, so V, =0. ©) The voltage reading over the switch is that over the battery: V, = 24.0V. d) Having closed the switch: £=24.0 V/388 Q= 4.08 A > V,, = 24.0 V - (4.08 A)(0.28 Q) = 22.9 V. ¥, = IR = (4.08 A)(5.60 Q) =22.9 V. ¥, = 0, since all the voltage has been “used up” in the circuit. The resistance of the switch is zero so V, = JR =0. 25.35: a) When there is no curreat flowing, the volimeter reading is simply the emf of the battery: € = 3.08 V. b) The voltage over the internal resistance is: ¥, =3.08 V-2.97Ve0nVves reo = 2 UY occa 165A c) Vy =297V=(.65A)R r-297N ea 65 A 25.36: a) The current is counterclockwise, because the 16 V battery determines the direction of current flow. Iis magnitude is given by: _=e_ 16.0V-8.0V = oe ___ VON O_o, TR 16045.0941404+9.00 b) Fy =16.0 V— (1.6 290.47 A) =15.2 V. c) V,, = 6.02047 A) + (1.4 QN0.47 A) +8.0V =110V. d) 25.37: a) Now the current flows clockwise since both batteries point in that direction: _é_ 16.0V+8.0V “ER 1604500414049.02 b) Fy = 160 4.6 Q)0.41 A) = 13.7. c) F,, = -G6.0 Q\L.41A)— (1.4Q)1.41 A) +8.0 V=-1.0V. a =141A. 16y 25.38: a) V, =19V>I=¥%,,/R, =19V/9.0 O=0.21A b) TE=UR > 8OV=(0.649.04144 HAO2MA)S Ras 26.10, c) 16y t " b a ‘ 7 25.39: a) Nichrome wire: 16.0 120 Vollage 40 00. 0.00) 1.00 2.00 ‘Current (A) b) The Nichrome wire does obey Ohm’s Law since it is a straight line. c) The resistance is the voltage divided by current which is 3.38 Q. 25.40: a) Thyrite resistor: Voltage wy) 0.00 1.00 2.00 3.00 4.00 ‘Current. (AY b) The Thytite is non-Ohmic since the plot is curved. c) Calculating the resistance at each point by voltage divided by current: 6.00 5.00 Resistance 4-0 (Ohms) 5 49 2.00 1.00 0.06 100 2.00) 3.00 4.00 Current (A) 4L: a) r=E/F=1.50V/14.8A=01012 b) r=€/£=1.50V/68A=0.220 c) r=8/£=12.6 V/1000 A= 0.01262. 28.42: a) P=V?/R> R=V"/P = (15 V) /327 W = 0.6880. b) a an BN =218A4 R 0.68.2 25.43: P=1T =(650 V)(0.80 A) = 520 W. 25.44: W = Pé= IV = (0.13 AN9 V)1.513600 8) = 6318 J. P 25.45: a) P=I°R=> p=——= vol E= al. b) From (a) p=J’p. AL c) Since J = E/p, (a) becomes p= E*/p. 7AR Ate A) £ =J* p= p=JE since 25.46: a) = DE/R,,,,, =8.0 V/17 Q=0.47 A= PB, =1"R=(0.47 A)'(5.0Q)= LI W and By =1°R =(0.47 AY (9.0 Q)= 2.0 W. b) Bey = EL - I’r = (16 V\(0.47 A) — (0.47 A)".6 Q)= 7.2 W. c) Py =€2 +4? = (8.0 V\(0.47 A) + (0.47 A)’ .4Q) = 4.1 W. d) (b)=(a)+©) 25.47: a) W = Pe = IVE = (60 A)(12 V(3600 s) = 2.59 x 10° J. b) To release this much energy we need a volume of gasoline given by: 6 p= PWS 56g vot = 0055 ke 46,000 J/e ep 900ke/m? c) To recharge the battery: #=(Wh)/P = (720 Wh)/(450 W)=1.6b. = 6.22107 mé = 0.062 liters. 25.48: a) £=E/(R+r)=12 V0 Q=1.2A P=El = (12 VI.2A)=M4 W. This is less than the previous value of 24 W. b) The work dissipated in the battery is just: P= Z*r = (1.2 4) (2.0Q)= 2.9 W. This is less than 8 W, the amount found in Example (25.9). c) The net power output of the battery is 14.4 W—2.9 W=11.5 W. This is less than 16 W, the amount found in Example (25.9) 25.49: a) [=V/R=12 V/6Q=2.0A= P= &1 = (12 V) (2.0 A)= 24 W. b) The power dissipated in the battery is P= Z*r = (2.0 A)?(1.0.)= 4.0 W. c) The power delivered is then 24 W—4 W = 20 W. 25.50: a) P= LE) R=3.0V/ITQ=0.18 AS P=PR=0,529 W. b) W = Pi= JV: = (0.18 A)(3.0 V)(6.0)(3600 s) = 9530J. c) Now if the power to the bulb is 0.27 W, 2 P=FRs027w=|—2_ (79) d7Q4 RY = 567? > R=682 17Q+R 25.$1:a) P=V?/R>R=V?/P = (120 V)?/540 W= 26.70 b) F=V/R=120 V/ 26.7 Q=4.5A. c) Tf the voltage is just 110 V, then J =4.13 A> P=VI=454 W. d) Greater. The resistance will be less so the current drawn will increase, increasing, the power. 25.52: From Eq. (25.24), p= ne'c 9.11107 ky 7 re = en 55 xt 0s, ne'p (10x10 m) (1.60 x 10 C}* (2300 Q- m) b) The number of free electrons in copper (8.5 x10" m™*) is much larger than in pure silicon (1.0 10" m~*), 3 95.53: a) p = RA O104 2) @/4) (2.50% 107m)" _ 5 65010 oom, E 14.0 m br 14-4. 028 Vinal 250x109" my a 3.65x10° Om J oe 1.28 Vim ©) Vs TT exo O mR axld eel nq png (3.65x10* Q-m) B.5x10" m*) 0.6 x10 C) = 2.58 %107 mvs, 2.00 cm T=0.100 mm As gf, _V_ Vv _VA VQ?) R pid pl pl _ (2 V) (22)(2.00 x 10 m) (0.100 x 10 m) - (14710 Q- m) (25.0 m) =410A 25.55: With the volumeter connected across the terminals of the battery there is no current through the battery and the voltmeter reading is the battery emf; s-12.6 V. With a wire of resistance R connected to the battery current J flows and ¢ —Ir-IR=0 Call the resistance of the 20.0-m piece R,; then the resistance of the 40.0-m piece is R, =2R, e-Lr-1R,=0, 12.6 V—(7.00 Ayr— (7.00 A)R, = 0 e—Lyr—1,(2R,) =0; 12.6 V— 4.20 A)r— 4.20 A)2R,)=0 Solving these two equations in pvo unknowns gives R, = 1.2002 This is the resistance of 20.0 m, so the resistance of one meter is [1.20°2/(20.0m)] (1.00m)= 0.0600 25.56: a) [ =—=——__ R Rath, and. 2 p,, Poko 072x107 2-08 Mm) _ 9 yoo Ao, (W/) (6.0% 10" my" and L ro. ny, = Pacha GAT AO™ Btn) .2) _ 9 969 Aug (x/4) (6.010 my xe SaN 45 A. “0.049 2+ 0.0622 So the current in the copper wire is 45 A. b) The current in the silver wire is 45 A, the same as that in the copper wire or else charge would build up at their interface. ©) Bogs Joey = Pot = CA OOO SD 9 76 ven ie i IR, (43. A) (0.0629) d) Ey, = Jpg, = = 2.33 Vm. ) Ese = Bag La 12m /m. €) Vig = ERy, = (45 A) (0.0622) = 2.79 V. 25.57: a) The current must be the same in both sections of the wire, so the current in the thin end is 2.5 mA. to. 3 W) Been pf BE = 72X10" Bm) 5A) _ 914 05 Vin A (2/4) 0.610 A)? (1.72 *10 Q-m)(2.5x10" A) (x/4) (080107 A)? =8.55 «10° Wim (= 4E, gn) d) ¥ =Esumlemm +E SV = (2.14107 Vim) (1.20 m) + (8.55 10° V/m) (1.80 m) =1.80x107 V. of °) Foam = pl =P = 08mm~o eum 25.58: a) —X_- of dre!) volume 2 K volume =8.7x10" J/m’, b) U=g¥ =ne(volume)V = (8.5% 10"m™) (1.6 x 10°C) 00% m’) (1.0 V) = 13600 J. And the kinetic energy in 1.0 em’ is K = (8.7% 107° Jim’) (10%m) = U__136003 8.7 x10" J. So = K 8.7x10%6 J =T@5 10” m1) (9.11. x 10 kp) (1.510 m/s)" 16x10". 25.59: a) “a =), ph A, gph 1 mm) ul, ® nn b) When pay =r, R= PE a A 4 ‘5 25.60: a) a= a FoR 2 Go) -£(-4} an anv, 4n\a b ral “idea jah Votead___Voab_ R p(b—a) A plb-adar? plb-ayr c) If the thickness of the shells is small, we have the resistance given by: -2(t-f)-a ph here L = b— 4c\a_b) 4nab "ea? a ° 28.61: E= pJ and E == Bo = pJ == AJ = 1 = 72; = leakage current. 25.621 a) 1=4$ 5 J =4= = pigy= F- So wo make the currentdensity a maximum, we need the length between faces to be as auall as possible, which means Z = d. So the potential difference should be applied to those faces which are a distance d apatt. This maximum cusent density i8 Jiggy = b) For a maximum current J =4= = = JA must be a maximum, The maximum area is presented by the faces that are a distance d apart, and these two faces also have the greatest current density, so again, the potential should be placed over the faces a distance @ apart. This maxinmm current is Va we =O ? 25.03: a) R= 2 dO OD ni Om) OD) costa A (7/4) (0.0016 m)? b) p(T)= pol + aAT) = p(60° C) = (9.5 x107 Q- mu) (1 + (0.00088(C°y") (40°C) = p(60°C)=9.83x10" Q-m= Ap=3.34 x10" O-m. c) AV =BV%AT = AAL = A(fLAT) > AL = fLoAT = (18 x10 (C2y") x (0.12 m) (40°C) > AZ = 8,64 107 m = 0.86 mm. The volume of the fluid remains constant, As the fluid expands the container, outward expansion “becomes” upward expansion due to surface effects. 4) R= PA apa Ph , PAL 4 4. A = ap - G34%10" @- m)(0.12m) (95 x10 - m) (0.86 x10" m) (@/4) (0.0016 m)* (@/4) (0.0016 m)* =2.40x10° @) From Equation (25.12), a=: (£-1)= aig eomaeneae’ay _ 7) 1.1107 (Cy. This value is greater than the temperature coefficient of resistivity and therefore is an important change caused by the length increase. 25.64: a) peden BOW AAOV. GA, SR 24.08 = F., =8.00 V — (0.167 A) (8.50.2) = 6.58V. b) The terminal voltage is Vi, = + 4.00V + (0.167 A) (0.50 2) =4 4.08 V. c) Adding another battery at point d in the opposite sense to the 8.0 V battery: paoé _104V-80V+40V MR 24.50 =F, = 4.00 V — (0.257 A) (0.50 Q)= 387 V. = 0.257 A, and so 25.65: a) V, =€ —Ir > 8.4V = - (1.50 A) and 9.4. V=E 4 B.50A)r 394 V= 8.4 V+ (1.50 Ayr) + (3.50 Air 500A b) & =84 V+ (1.50 A) @.20Q)=8.7V. 25.66: a) F=V/R=14 kV/d0kQ+4+ 2kQ)=1.17 A. b) P-2°R- .17A)? 10,000 &) - 13.7 KW. c) If we want the current to be 1.0 mA, then the internal resistance must be: 14,0000 _ 1.410’ R=14 MOQ-10 kw 4 MO. 25.67: a) R= LE = GOS O10B) _ 999.0 4A (0.050 my b) ¥ = JR = (100 x10A) (1000 Q) = 100. c) P-=VI = (100V) (100% 107A) =10 W. 25.68: a) V = 2.50! +0.3601? = 4.0 V. Solving the quadratic equation yields £=1.34 Aor 8.29 A, so the appropriate cusrent through the semiconductor is Fa134a b) If the current J = 2.68 A, =F = (2.50 V/A) (2.68 A)+ (0.36 V/A?) (2.68 A)? =9.3V. 25.69: V =IR+V (1) =IR+ a + f° =(a4 R)E+ Bf = pl’ +(R+a)I-V=0 3 (1.3) 4+G843.2)1-12.6=057=142 A & 786V _ggsaa =o = TV _ oan, 9.25 A Rt+r 08504240 b) BI? + (a+r) I-€=0= 036" + (2.50 +085) -786=0 SI=194A c) The terminal voltage at this current is Vi, =€ - Ir =7.86 V — (1.94 A) (0.85 Q)=6.21 V. 25.70: a) r= 25.71: a) With an ammeter in the circuit: =—* _ 6a Lr +R+R,). r+R+R, ar a) So with no ammeter: é re 1 [AA =r (1+ 8) r+R- r4+R r+R, b) We want: 4 [144+ )1.015 4 0.01 R, (0.01) (0.45 2+ 3.8.2) dy r+R r+R 0.04250 ©) This is a maximum value, since any larger resistance makes the current even less that it would be without it. That is, since the ammeter is in series, ANY resistance it has increases the circuit resistance and makes the reading less accurate. 25.72: a) With a voltmeter in the circuit: £_sy,-e-4=e@/1-—1_|. R r+R, r+R, b) We want: Ya _[;-* _}0.99 «0.01 é r+R, r+R, = RL TOOW" _ 99, 99.45. = 44.60. 0.01 c) This is the minimum resistance necessary—any greater resistance leads to less current flow and hence less potential loss over the battery’s internal resistance 25.73: a) The line voltage, current to be drawn, and wire diameter are what must be considered in household wiring. P_ 4200 W b) P=Wiai= =35 A, so the 8-gauge wire is necessary, since it can vo 120V cary up to 40 A. ©) P=FR= Peb.. GSAY' G.72410" Bm) 420m) _ 196 yy (a4) (0.00326 m)* d) If 6-gauge wire is used, 1 Pph _QSAY (1.72107 Q-m) Bm) _ ge wy A (@/4)) (0.00412 my" = AE = APt= (40 W) (365) (12 h) =175 kWh = Savings = (175 kWh) ($0.11/kWh) = $19.25. pai’ 25.74: Initially: Ry = V//Iy = 20V)/(.35 A)=88.92 Finally: R, = V/L, = 120 V)/0.23 A)= 97.6 Q And Seisam,-—) 50-8) = 1 (#1) maw Z2- ) R R 45x10 °C |38.99 2, -f=2TPCST, =UPC+ 20°C = 237°C, b) Gi) Fy = 7) = U20-V) U.35 A) = 162 W (ii) P, =VE, = (120 V)(1.23. A)= 48 W @ 95.75: a) [= 2E. OV -80V ZR 10.02 d) Poy =L? Roy = (40 AY 10) =1.6 W. c) Power generated in €,, P= &, I = (12.0 V) (0.40 A)= 48 W. d) Rate of electrical energy transferred to chemical energy in &, P= E,L = (8.0 V)x (O40 A)= 3.2 W. e) Note (c) = (b) + (d), and so the rate of creation of electrical energy equals its rate of dissipation. =040A. pL _(2.0%107Q-m) 2.0m) 25.76: a) Rug = 2 = a 3157x107 2 A (=/4) (0.018 my z -2 2g, -HE GRA 2-m B5m) _ 917.6 a (/4) (0.008 my? SV =IR=L (Rog + Ro, )= (15000 A) (1.57 x10 24 0.012 Q) = 204 V. eet b) B= Pr=J° Re = (15000 A)’ (0.0136 &) (65x 10 s)= 199 J. 15.77: a) UF =ma=|q/E> 41-4, m lal _ ab m b) If the electric field is constant, %,, = EL = 2+ a ©) The free charges are “left behind” so the left end of the rod is negatively charged, while the right end is positively charged. Thus the right end is at the higher potential ¥,, 1.010% V) (1.6 x10" © a) a= Mell GO%107 VY) E6410 CY 510 mist mL (9.11107 kg) (0.50 m) ¢) Performing the experiment in a rotational way enables one to keep the experimental apparatus in a localized area—whereas an acceleration like that obtained in (@), if linear, would quickly have the apparatus moving at high speeds and large distances. 25.78: a) We need to heat the water in 6 minutes, so the heat and power required are: Q= me, AT = (0.250 kg) (4190 Jkg°C) BO°C) = 83800 J p= 2 838001 assy. ¢ 6608) 2 a But pat sor-F Sue = 6182. ‘P B3W =a by RaLER oe Revol. [GL8Q) C:5%10" m) ayy d 1.00%10° Q-m Now the radius of the wire can be evista from the volume: 1 2.5x107 m? vol= E(art)=> r= PPO = JSP 8g 5 104 me a (39 m) 25.79: a) V,, = -Ir=12.0 V- (-10.0 A) (0.24 Q)= 14.4 V. b) E= Pt=IVi = (10 A) (14.4 V) (5) (3600 5) = 2.59 x 10° J. ©) Engg, = Pagt = 1° rt = (10 A)’ (0.24 Q) (5) (36008) = 4.32 «10° J. d) Discharged at 10 A: fe = pn ET 120V- G0 A) 0.24) r+R Tr 10A, e) E=Pt=IV¥i=(10 A) (9.6 V) (5) (3600) = 1.73% 10° J. f) Since the current through the internal resistance is the same as before, there is the same energy dissipated as in (¢): Ey, = 4.32 x10° J. = 0.96.2 g) The energy originally supplied went into the battery and some was also lost over the internal resistance. So the stored energy was less than was needed to charge it. Then when discharging, even more energy is lost over the internal resistance, and what is left is dissipated over the external resistor. 25.80: a) V,, = € —Jr=12.0 V- (30 A) (0.24.Q)=19.2V. b) E= Pi=IVi= GOA) (9.2 V) .7) 3600s) = 3.53% 108 J. ©) Egy = Phgt =I°R¢ = (30 AY? (0.24 Q) (1.7) (36008) =1.32 «10° J. d) Discharged at 30 A: po pS HH 12.0V- BOA) 0249) _yigg r4R I 30A e) E= Pt=IRt= (30 A)? (0.16 Q) (1.7) (3600) =8.81 x10" J £) Since the current through the internal resistance is the same as before, there is the same energy dissipated as in (c): E,, =1.32x10° J g) Again, the energy originally supplied went into the battery and some was also lost over the internal resistance. So the stored energy was less than was needed to charge it. ‘Then when discharging, even more energy is lost over the intemal resistance, and what is left is dissipated over the extemal resistor. This time, at a higher current, much more energy is lost over the internal resistance. i: Age) __ ig nal Go. myn = 25.81: a) @ (2) Pop y= Ine) =F. P b) w=-aF =-(—5x107 K)') 293 K)=0.15. p= a= if" = 35x10 Q-m) (293K =8,0%10 O-m-K, c) P=-196°C=77K:p Sereno” Qm. 3 P= —300°C=573K 1p = eee = 3.99 10" Oem, 25.82: a) € = IR+IR, > 2.00 V = 10.09) + ¥ 2 =Llexp(ev /AT)- 140. b) £, =150%107 A, T =293 K 1333 = exp [39.6 V — 667] + 667 V. Trial and error shows that the right-hand side (chs) above, for specific V values, equals 1333 V, when ¥ = 0.179 V. The current then is just £=E, exp[39.6 V -1]= (1.5107 A) exp [39.6 (0.179) -1]=1.80 A. Pax _ Py expl=x/L] dx 25.83: a) R= Fo ar= 4 4 = R= exp [-x/L]de 9 8 hg p= 88 aysspet he = 8-2 Lexpl-a/il =A 0-1-2 alte aur) ait ett b) A(x)= = ONES: e 2, Be) a Th) Ket o) V@=% —=-+ CV (0)=%, = +05 C=— "d-e") oo dee) ide") ara € d-e) d) Graphs of resistivity, electric field and potential from x = 0 to Z. stakes : ae DBD and. elo + 1.24 Mase Resisiivity MO > * Heetrie field ha ae sm OS oo Te) a) 0 oetia 040) oan om PM ow, - 25.84: a) F=—©_ = Poet fr 3 <6 — 2 = 0 for maximum power output. r+R d b) For the maximum power output of (a), pea jos 4R- Wr R=r. r+ % Then, P=/?R= (2) ar a 26.1: a) R. -($+ a =12.32 = la7* 20 b) le 240V ios a. R, 1238 V_240Vv_ Vina = B= Gog 7 TS Aiton = 1A 1.1) (R+R) RR, 2.2: R,=|—-+—-| =|2=* ate 4s aa) (A) TAREE Ry >R,=R, ¥,; false. bh) true. 26.4: a) False, current divides at junction a. b) True by charge conservation. ©) Tme, Vir V sol ok d) False. P=IV.K, =¥,, but J, #I,,30 B#B,. e) False. P=IV =. Since R, > RP, < R. f) True. Potential is independent of path. g) True. Charges lose potential energy (as heat) in R,. h) False. See answer to (g). i) False. They are at the same potential. 26.5: a) a-(aa —* asl =082. 24Q 162° 489, B) Zp =6/ Rog = 8 V)/(2.4 Q)=11.67 A Lg = e/R,, = (28 VYIU.6 Q)=17.5 Lag =e[Ryg = Q8 V4.8 Q)= 583A. Loa: = &/ Bory = 8 V){(O8 Q)=35 A. d) When in parallel, all resistors have the same potential difference over them, so here all have V= 28 V. °) 8) Bg HL’ Roy = (11.67 A)? (2.4 Q)= 327 Wi Ag =I Rg = ATSA) 6 Q)= 490 W; P,, =I°R,, = (8.83 A)’ (4.8 Q) =163 W. f) For resistors in parallel, the most power is dissipated through the resistor with the 2 least resistance since P= J?R= 5 with V = constant. 26.6: a) RUHL KR =2424+1.6244.8Q=882Q. b) ‘The current in each resistor is the same and is f= = 8 _ 318.4, R, 882 ¢) The current through the battery equals the current of (b), 3.18 A. d) Meg = IR,4 = G18 A24 Q) = 7.64 Vig = Ry 5.09V5¥,, =1R,,= G18 AV4.8.Q)=15.3V. e) Py = PR, =G.18 A)(24 Q)=24.3 W: PB, 16.2 Wy Pig = E?Ryy = (3.18 A)?(4.8 Q) =A8.5 W. £) For resistors in series, the most power is dissipated by the resistor with the greatest resistance since P —Z?R with J constant. 3.18 A)(.6 Q)= 6 = B18 AP d.6Q)= 2 26.7: a) P= > = V =\PR = (6.0 W)(15,000 Q) = 274 V. v?_ a2ovy b) P=—= R 9,000.2 =16W. ( a “1 Hae =5.009. (saa 6.002 12.08 4.002 Loom *1Reu = 6 00 V}/(5.00 2) =12.0A Li, = (12.0) = 3.00 A; F, = ——— (12.0) = 9.00 A; " a D ‘ i » 6 3 F, =—— (12.0) = 8.00 A; 7, = (12.0) = 4.00 A. 3460) oe 1 1 ———=— 00 24+1.0082 5.00 £24 7.00 22 = 6/ Ry = (48.0 V)/B.00 Q)=16.0 A. Vota 41 26.9: } = 3.00.2. Lot 4 ——~(16.0)= 4.00 As Z, = Gap 160)= 4.00 Aid, 12 =——_(16.0)=12.0A. Fe) 26.10: a) The three resistors Rp, Ry and R, are in parallel, so: 4 Ruy = oadwds. = ee =0.992 R RR, 8.208 1.500 4.500 = R= Rt Ry = 3.5004 0.992=4490. = _ 6.0V_ =134A5/% =, = (1.34 A) (3.50 Q) = 4.69 V. R, 4490 = V pq, =E.Rosy = 1.34 A) (0.99 Q)=1.33V SF, = 133 _ 0162 A, ‘ing =A Raa = . - 2 E2007" . ¥, By 133V _ 9996 a. Q b) 2 =2)R, = (1.50 A)?(4.50 2) = 10.13 W, P, = B =P, -h = 1.125 W. ¢) If there is a breakat R,, then the equivalent resistance increases: 4 “1 R,=RtRy=R+(t4+) -4500+(2 = 6.75. Ry R 4.502 And so: 2 _9.00V 6.75.2 =133A,f, =L=5h = 0.667 A d) B =2'R, = (1.33 A)'(4.50.Q) =7.96 W, PB, = FAH 199W. e) So &, and R, are brighter than before, while R, is fainter. The amount of current flow is all that determines the power output of these bulbs since their resistances are equal. 26.12: From Ohm’s law, the voltage drop across the 6.00 Q resistor is ¥= IR = (4.00 A)(6.00 2) = 24.0 V. The voltage drop across the 8.00 @ resistor is the same, since these two resistors are wired in parallel. The current through the 8.00 © resistor is then J =7/R= 24.0 V/8.00 © = 3.00 A. The current through the 25.0 © resistor is the sum of these two currents: 7.00 A. The voltage drop across the 25.0 @ resistor is 7= IR = (7.00 AX 25.0 &)= 175 V, and total voltage drop across the top branch of the circuit is 175+ 24.0 = 199 V, which is also the voltage drop across the 20.0 Q resistor. The current through the 20.0 Q resistor is then J = 7/R =199 V/20Q=9.95 A. 26.13: Current through 2.00-Q resistor is 6.00 A. Current through 1.00-@ resistor also is 6.00 A and the voltage is 6.00 V. Voltage across the 6.00-Q resistor is 12.0 V + 6.0 V= 18.0 V. Current through the 6.00- resistor is (18.0V)/(6.00) = 3.00 A. The baitery voltage is 18.0 V. 26.14: a) The filaments must be connected such that the current can flow through each separately, and also through both in parallel, yielding three possible current flows. The parallel situation always has less resistance than any of the individual members, so it will give the highest power output of 180 W, while the other two must give power outputs of 60 W and 120 W. 2 3 4 s cow= Rp = 9%) _ goo and wow=1-3 z, -8 _ i900 R cow RE 120 W v a20vyt__ a2ovyt Check for parallel: P =180W. Gte) Giat na) 802 b) If R, burns out, the 120 W setting stays the same, the 60 W setting does not work and the 180 W setting goes to 120 W: brighinesses of zero, medium and medium. ¢) If R, bums out, the 60 W setting stays the same, the 120 W setting does not work, and the 180 W setting is now 60 W: brightnesses of low, zero and low. 120V (400 2 + 800 2) b) Bog = LR = (0.100 A)?(400.2) =4.0 W; Puy =?R = (0.100 A}(800 ) = 26.15: a) I= =0.100A 8.0 W => Pay =4W48 W=12 W. c) When in parallel, the equivalent resistance becomes: “a Ry wef bye =26705 1-2-2 aig a (4008 * 8008 R, 2678 800 4 Bigg ® — 400 __ 6.449 Ay= 0.150. 400 +800 400+800 d) Poy = ER = (0.30 AJ*(400 0) = 36 W; Poy = LR = (0.15 AY*(8002) =18 W (0449 A) = 0.30 A; Lang = = Pyqy = 36 W418 W = 54 W. e) The 800 & resistor is brighter when the resistors are in series, and the 400 Q is brighter when in parallel. The greatest total light output is when they are in parallel. ¥ av)? ve aovy 26.16: a) = = = 20 25 Reggyy = = ) Reow = ew mw Pp 200W or <1. -£- Mov ae OR (24024722) = 72 = 0.769 A. b) Prog = L°R = (0.769 A}? (240 Q) = 142 W; Pow = LR = (0.769 A)?(72.Q) = 42.6 W. c) The 60 W bulb burns out quickly because the power it delivers (142 W) is 2.4 times its rated value. 26. oo oot 00 ne ANAS ARAN = wy = on AANA 30,0 V—Z (20.0245.004+5.00)=0, [=100A For the 20.0-Q resistor thermal energy is generated at the rate P=PR=200W. Q= Pi and Q = mcAT gives a MeAT _ (0.100 kg) (4190 J/kg K) (40.06%) 0110's P 20.0 W 26.18: a) PHLR, 20 W = (2A)?R, > R, = 5.002 R, and10Q in parallek (10 Q)Z,, =(5Q)2 A) Tip =1A So fF, = 0.50 A. R, and R, are in parallel, so (0.50 A)R, = (2A) (SQ) R, =2000 b) ©=% =(@A)5.Q)=10.0V c) From (a): I, = 0.500 A, J, = 1.00 A 4) A =20.0 W (given) % Fi if Ry, = (0.50 A)’ (20 Q) = 5.00 W 9 =H Ry = 0.0 A)? 0.2) = 10.0 W Proig = 20 W +5 W410 W = 35.0 W Prater, = LE = 3.50 A) (10.0 V) = 35.0 W Prosi = Panes, Which agrees with the conservation of energy. 26.19: a) I, =6.00A-4.00A = 2.004, b) Using a Kirchhoff loop around the outside of the circuit: 28.0 V - (6.00 A) (3.00. Q)- (2.00 A) R=0= R= 5.002. ©) Using a counterclockwise loop in the bottom half of the circuit: — (6.00 A) (3.00.2) — (4.00 A) (6.00 2) = 0 6 =42.0V. d) If the circuit is broken at point x, then the current in the 28 V battery is: _Ze_ 28.0V = => =3.50 A. IR 3.00245.002 26.20: From the given currents in the diagram, the current through the middle branch of the circuit must be 1.00 A (the difference bebween 2.00 A and 1.00 A). We now use Kirchoff's Rules, passing counterclockwise around the top loop: 20.0 (1.004) (6.00 A 11.002) 1 (1.00 A)(1.00211.002) 4-0 o,-18.0V. Now traveling around the extemal loop of the circuit: 20.0. V- (1.00 A)(6.00 Q +1.00 ©)- (2.00 A)(1.00 24 2000)- «, =06,=7.0V. And V,, = (1.00 A)(4.00 2+ 1.00 ©)+ 18.0 V = 413.0 V, s0 %, =-13.0V. 26.21: a) The sum of the currents that enter the junction below the 3-Q resistor equals 3.00 A+ 5.00 A= 800A. b) Using the lower left loop: ,— (4.00 2)(3.00 A)— (3.00 2)(8.00 A)=0 = = 36.0V. Using the lower right loop: , — (6.00 &)(5.00 A)— G.00 2)(8.00 A)= 0 >, =54.0V. ©) Using the top loop: 54.0 V—R(2.00 A)-36.0V=0> R= — = 9.00.2, 26.22: From the circuit in Fig. 26.42, we use Kirchhoff’s Rules to find the currents, £, to the left chrough the 10 V battery, Z, to the right through 5 V battery, and Z, to the right through the 10 © resistor: Upper loop: 10.0 V —(2.00 243.00 Q)z, — (1.0024 4.00 Q)Z, - 5.00 V=0 = 5.0 V—(5.00 OY, - (5.002), =05 1,4 £,=1.00A. Lower loop: 5.00 V + (1. masa (16.0.2), =0 = 5.00 V+(5.00 QY, —(10.0. 2), =0 > 1, -22, = Along with Z,=Z, +2, we can solve for the three currents and find: = 0.800 A, Z, = 0.200 A, I, = 0.600 A. b) ¥, =—(0.200 A)(4.00 0)— (0.800 A)(3.00 2)= -3.20 V. 1.00 A, 26.23: Afier reversing the polarity of the 10-V battery in the circuit of Fig. 26.42, the only change in the equations from Problem 26.22 is the upper loop where the 10 V battery is: Upper loop: 10.0 V — (2.00 © +3.00 Q)é, — (1.00.24 4.00 QV, - 5.00 V =0 = -15.0V— (5.002), -(S00QV, =0>5 1 +1, =-3.00A. Lower loop: 5.00 V + (1.00. Q+ 4.00 2), - (10.0 Q)z, =0 = 5.00 V+ (5.002, - (10.02), =05 7, -2f,=-1.00A. Along with I, =, + £,,we can solve for the three currents and find: I, =-1.60A, I, =-140A, J, =-0.200 A. b) Vy =+(.40A)(4.00 2)+ (1.60 A}(3.00 2)=10.4 V. 26.24: After switching the 5-V battery for a 20-V battery in the circuit of Fig. 26.42, there is a change in the equations from Problem 26.22 in both the upper and lower loops: Upper loop: 10.0 V — (2.00 2+ 3.00 Q)F, - (1.0024 4.00 2), 20.00 V =0 10.0 V- (5.00 O), — (5.00 Q¥, = 05 L,4+2,=-2.00A. Lower loop: 20.00 V + (1.00.24 4.00 Q), - (10.0. Az, = 0 = 20.00 V + (5.00 Q)F, - (10.0 QV, = 0= F,-24, =-4.00A. Along with J, = Z, + Z,,we can solve for the three currents and find: Z,=-O04A,f, =-1.6A,f, =41.2A. b) £,(4Q)-1(3.Q)= (1.6 A)(4Q)+ (0.4 A\3.Q)-7.6V 26.25: The total power dissipated in the four resistors of Fig. 26.10a is given by the sum of: P= PR, =(0.5A)(2Q)=0.5 W, B= 2? R= (0.5 AY BQ)=0.75 W, =PR,=(05A)(4Q)=1W, BP =P? R, =(0.5 AV (TQ)=1.8 W. = Poy = Bt Rt +P =4W. 26.26: a) If the 12-V battery is removed and then replaced with the opposite polarity, the current will flow in the clockwise direction, with magnitude; pee VHA ER 162 b) ¥, =-(R, +R, +e, =-(4 O47 Q)IA)+4V=-7V. =1A. 26.27: a) Since all the extemal resistors are equal, the current must be symmetrical through them. That is, there can be no current through the resistor R for that would imply an intbalance in currents through the other resistors. ‘With no current going through R, the circuit is like that shown below at right [=aa OD, © MN an X10 wy ye og on Mie an pS rasa \ By Tay > « By So the equivalent resistance of the circuit is = FY 951, BY asa 22 22 12 liad = Fan =6.5A, and no current passes through R. b) As worked out above, R,, =12. ¢) ¥, =0, since no current flows. d) R does not show up since no current flows through it. 26.28: Given that the full-scale deflection current is 500 A. and the coil resistance is 25.00: a) Fora 20-mA ammeter, the two resistances are in parallel V,=V, => LR, = LR, > (50x10 A\25.0.0)= (20107 A—500x10° AJR, => R= 06410 500 jx, b) For a 560-m voltmeter, the resistances are in series: Yap i Vg =H(R,+R)>R, > 3 , = 30010 VY 95.00=9750 500x10° A Vin=500MV a 26.29: The full-scale deflection current is 0.0224 A, and we wish a full-scale reading for 20.0 A. (0.0224 A)(9.36. + R)= (20.0 A - 0.0224 A)(0.0250) = 2A OA. os6o 199° 0.0224 A Rg=9.36Q AC.0224 4 f+ « 20.04 R,= 0.0250 26.30: ayr=—2- = VY _ ooo a Rom (8.234+425Q) => PV =e-Fr =90V -(0.208 A)(8.23Q)= 88.3.0. er __ eR, b) Vse-=e- r+R, r+R, Now if Vis to be off by no more than 4% it requires: 26.31: a) When the galvanometer reading is zero: Ry x oy = IRy and 2, =IRy =e = 4 9% = 65 be b) The value of the galvanometer’s resistance is unimportant since no current flows through it. co) 2, = (9.15 Va 26.32: Two voltmeters with different resistances are connected in series across a 120-V. line. So the current flowing is 7= = _120V___y 99.10 A. But the current Ryo 100x107 O required for full-scale deflection for each voltmeter is: 150V 150V 3 iB =?" _ 0.0150 Aand = 8167x107 AL F400 EO) 19 999 #449) ~ 99 900 0 * So the readings are: 1.20x10° A 1.20x10° A Vou =150°W) —— —** |= 12 Vand % ,. = 150 V) > |= 108 Vv. 108 ( 0.01504 } ee fades 26.33: A half-scale reading occurs with R = 600 2. So the current through the galvanometer is half the full-scale current. -3 26H F Ry 2150 V=[22I0"A isons stoa+, >A, =2180 26.34: a) When the wires are shorted, the full-scale deflection current is obtained: 2 = Reg, = 1.52 V= (2.5010 Al 65.024 R)> R= 543.0. Veta b) Ifthe resistance 8, -2000:¢-—" -___52V a gg ma, Rog 65.024543 04K, OL _ 1.52V =x -12Y_ gogo, Roy 65.004 543.04 RK, E, Ig = 6.25210 A= R= —E2V __ gogo -18 0. .25%107 A 1 = jg H1.25%107 A = R= 2) __ 608 2 = 608.0 2 125x107 A 1.52V Taso A OR OHO 3 gs Epo Flgg =18TSHIO? AZ Ry = 26.35: [RC]= [F2] = (2] : [2] -(] 26.36: An uncharged capacitor is placed into a circuit. a) At the instant the circuit is completed, there is no voltage over the capacitor, since ithhas no charge stored. b) All the voltage of the battery is lost over the resistor, so V, = ¢ =125 V. ¢) There is no charge on the capacitor. d) The current through the resistor is 7 = ay = 0.0167 A. Rog 75002 e) After a long time has passed: The voltage over the capacitor balances the amf: V7, =125 V. The voliage over the resister is zero. The capacitor’s charge is ¢= Cy, = (4.60 x 10 F)(125 V)= 5.75 «107 C. ‘The current in the circuit is zero. ce ee 6.55x10" C RC (28x10 Q)(4.55x10" BF) by T= RC= 0.28% 10° &)(4.55 «107 F) = 5.82107 s. 2637: a) i =112%104 A. 26.38: verve oC 4.00s =— = ig. a9 107 Ring!) (340% 10° Q) (in 12/3) 26.39: a) The time constant RC = (0.895 x 10° Q) (12.4x10¢ F)=11.1s.So at: t= 08: ¢=Ce—e"*")=0. t= 58: @=Ced —e"*") = 124 x10" F) (60.0 V) d—-e Cy =2.70x10" C. £=108:9 =Ce(l—e"'*°) = 12.4. x10" B) (60.0-V) de Cen) =4.42%104 C. t=208:g = Cele") =(12.4 x10 F) (60.0 V) I Oy 621x107 C. £=100s:q=Ce(l—e"*") = (12.4 «10° F) (60.0 V) de CY) =744x107C. b) The current at time ¢ is given by: i=—e"""°.So at: t=0s: = eS 0100107 A £2 $328 Oo AT A 95% 60.0 V 8.95x10Q 60.0 V 8.95x10° Q 60.0V 895x10 A t=10s:i= et 227x107 AL t=20s:i PMN SL ALxX10 A #=100s: ON 2820x107 A. ¢) Charge against time: me | “icc 400 se oo 2 Current against time: 26.40: a) Originally, r= RC = 0.870 s. The combined capacitance of the two identical capacitors in series is given by en owe G& CC € ‘The new time constant is thus X (¢) = 2°! =0.435 8 b) With the two capacitors in parallel the new total capacitane is simply 2 C. Thus the time constant is K(2C)= 2(0.8708) = 1.745. 1 1,1 2 c hoa = 2 WAL: 6 -Vy—Vo =0 £ =120V, ¥, = ZR = (0.900 A) (80.02) = 72V, soF = 48 V Q=C¥ = (4.00x 10°F) (48V)=192 pC) 26.42: a) Q= CV = (5.90% 10° F)(28.0 V)= 1.65 x10 C. b) g=O0-e") ee After #=3x107s:R=— = * Saggy (5.90 x10" F) (In(l - 110/165) c) Ifthe charge is to be 99% of final value: q =I RC fad-e"™)s t= ° RC Inl—¢/Q) =~ (463 Q) (5.90 10° F) In(@.01) = 0.0126 s. 26.43: a) The time constant RC’ = (980) (1.50 10 F)= 0.0147 s, 1= 0.058 :¢ =Ce(l— 6 *°) = (1.50% 10 F) 08.0 V) Ie 4) =1.33 x1 07 €. by re Settee 18.0V oreo 9305103 A R 9802 =F, =IR=(9.30x 10° A) 980Q)=9.11 Wand ¥, =18.0V—9.11 V=8.89 V. c) Once the switch is thrown, ¥, =V_ =8.89 V. er t= sig=Qhe =(1.50x . je =6.75% : d) Aft 0.013:q¢ =Qye* = (1.50% 10 F) (8.89 Ve = 6.75 x10 C. P4100 W 26.44: a) I= =17.1A. So we need at lest 14-gauge wire (good up to 18 20V A). 12 gauge is ok (good up to 25 A). wv @aovy P4100 2 bv) P= are R c) Atl1¢/kWhr = in 1 hour,cost =(11@/kWhr) (1 hr )(4.1kW) = 45¢. 26.45: We want to trip a 20-A circuit breaker: =1500W PF 8, with P= 900w: r= DOO , 909 W og TOV * 120V DOV” 120 26.46: The current gets split evenly between all the parallel bulbs. A single bulb will 90 W 204 draw f=? = 29% _ 0.75 4 Number of bulbs < = 267. So you can attach ¥ 120V 015A 26 bulbs safely. 16.47: a) P= = V6.9. a Pam =(6.0A) (120 V)=720 W. R202 b) At T= 280°C, R= R, (1+ aAT) =20 (142.8 x10 (C°yt (257°C) =3449. fh =3.49A => P= (3.49 A) 120 V) = 419 W. R 344A 26.48: a) tt » a,-(—_-44] - B&+%) SU URG+R, R R+R, +R, TER, =R RR +R, +R )= RR, +R) RAR (RAR VR. 26.49: a) We wanted a total resistance of 400 Q and power of 2.4 W froma combination of individual resistors of 400 Q and 1.2 W power -rating. b) The current is given by: f = VP/R = [2.4 W/400Q = 0.077 A In cach leg half the current flows, so the power in each resistor in each resistor in each combination is the same: P =(£/2)' R = (0.039 A)? (400 2) = 0.6 W. 26.50: a) First realize that the Cu and Ni cables are in parallel. B Ry = pb! A= pyw—y nal i Roy = Poy LIA = Pose? ay xa’, n(b! ~ a) Roe Pri Powe (4 Me “) Llp Pow x [cose 46 ony Gos e?) So 20m | 78x10" Gm 1.72 %10* Om Rese =13.6 10% O=13.6 pQ L L >) R= pe S= Pas Se _ xb? R_ x(0.10 m)’ (13.6 x10 Q) i 20m =2.14«10° Om 26.51: Let R= 1.00, the resistance of one wire. Each half of the wire has R, = R/2. The equivalent resistance is R, + R,/ 2 +R, =5R,/2=4(0.500.2)125Q 26.52: a) The equivalent resistance of the ewo bulbs is 1.0 2. So the current is: po = — i =4.4A > the current through each bulb is 2.2 A. Rew 10240808 Fug =e — Ir =8.0V - (4.4 A) (0802)=4.4 V5 B, =I =(22 A) (4.4 V)= 99 W b) If one bulb burns out, then po 8.0V Raw 2.0240808 =29 A= P=PR=(2.9 AY’ (2.0.Q)=163 W, so the renuining bulb is brighter than before. 26.53: The maximum allowed power is when the total current is the maximum allowed. value of = V/P/R = /36 W/2.4 Q =3.9 A. Then half the current flows through the parallel resistors and the maxinum power is: Py, =(L/2) R+ Z/2VR4 EPR =31R-3094) QAQ)=S4W. 1 1 1 stata] = 400; 8Q 162 162 26.54: a) RB, 16, 16)= ( fi 2 1) R(9,18)=| —~- +] = 602 nee (Ga*wal So the circuit is equivalent to the one shown below. Thus: 1 1 62462 0H+4Q 1 } =800 b) If the current through the 8-Q resistor is 2.4 A, then the top branch current is (8,16, 16)=2.4A+42.4A442.4A =4.8 A But the bottom branch current is twice that of the top, since its resistance is half. Therefore the potential of point a relative to point x is V,, =— IR,(9,18) =— (9.6 A) (6.00)= - 58 V. + Circuit (a) The 75.0Q and 40.0 Q resistors are in parallel and have equivalent resistance 26.09 Q The 25.0. and 50.0 & resistors are in parallel and have equivalent resistance16.67 Q. The network is equivalent to cua. "Fama ] nia vi A AANA soos APPR 09 1 a =18.7Q 100.08 23.050 °° Rs Circuit (b) The 30.0 Q and 45.0 Q resistors are in parallel and have equivalent resistance 18.00. The network is equivalent to 1 ow]. we | SAAN sone ARM on i a te > FHEO 10.0.2 AAAS AAA 2330 a | WAN 1 il 1 = + R, 1002 3032 80 R,=752 26.56: Recognize that the ohmmeter measures the equivalent parallel resistance, not just x roi, a aya 2022 ¥ 1150" 130Q 852 X= 4682. 26.57: Top left loop: 12 ~ S(Z, —4,)— IZ, =O = 12-61, +54, Top right loop :9 - 8(, + £,) IZ, =0-> 9-92, -81, =0. Bottom loop: 12—10£,-9+1Z,-Z, =0=3+2,-I,—102, =9. Solving these three equations for the currents yields: 2, = 0.848 4, f,=2.14 A and £, =O.171 A. 26.58: Outside loop : 24 — 7(1.8) - 3.8—1,)=0£, =-2.0A. Right loop: ¢— 7(1.8)- 22.0) =0 «=8.6V. 26.59: Left loop: 20-14 - 2, + 4(Z, -1,)= 0-9 6-62, +42, =0. Right loop : 36 — SZ, —4(Z, —1,)= 0 => 36442, -9f, =0. Solving these two equations for the currents yields: £,=S.2MA= Lig, 1, = 632A =Log,and Lg =1,- 1, =111A. 26.60: a) Using the currents as defined on the circuit diagram below we obtain three equations to solve for the currents: Leftloop: 14-4 -2(,-Z))=0 => 3A, -20,=14. Toploop: -2(/-f,) +h +4, =0 326435 4+£,=0. -(-£L4+h)+ AL -£)-£, >1+3f,-4f,=0. Bottom loop : Solving these equations for the currents we find: £= Lye = 10.0 ASL = fy, = 6.0 A; = So the other currents are: iy PHN =40 AGE, 31-1) = 40 AT, =I 4) = 6.0, 14, b)R 0 a = = HOY 1600 26.61: a) Going around the complete loop, we have: Ye- YR=12.0V-8.0V-19.02)=0= 1= 0.44 A. = V4 = Se - Y= 12.0 V-10.0V - (0.44 A) 20419419) =+0.22 V. b) Tfnow the points a and b are connected by a wire, the circuit becomes equivalent to the diagram shown below. The two loop equations for currents are (leaving out the units): 12-10-41, +42, =0= 1,=1,-0.5 and. 10-84, —5f,= 2-42, -(h +,)=0 = 2- (Al, -2)-5f,- SI, +25=0 31 = 0464, ‘Thus the current through the 12-V battery is 0.464 A. Gs a0 26.62: a) First do series/paralle] reduction: 209 “N 4 152 5 284 5 vin eS 209 j Kt e L Now apply Kirchhoff’s laws and solve for ¢. = (20 QZ A)-5 V-(20 QY, =0 DIA 44-24 31,=2A-(C225A)=425A AV tot 2052) (4.25 A)4 e— (20Q)(-2.25 A)=0 109 V; polarity should be reversed. b) Parallel branch has a 10Q resistance AV,,, = RE = (10.2) @A)=20V ii . pam 2v_ 2 Current in upper part: =“ = FS =3A Pt=U3P Rt=U > G 4) (102 = 60 i=1358 26.63: ona 20a naoy ova = os a. V, +1,0.0Q)+ 12.0V =F, V,-¥,=12.106V; ¥,-¥, =¥,-¥, =12.7V 26.64; First recognize that if the 40 © resistor is safe, all the other resistors are also safe. PR=P 3 P(40Q)=1W 1=0.158 A Now use series / parallel reduction to simplify the circuit. The upper parallel branch is 6.38 Q and the lower one is 25 Q. The series sum is now 126 Q. Ohm’s law gives &= 126 QX0.158 A)=19.9V 26.65: The 20.0-2 and 30.0-22 resistors are in parallel and have equivalent resistance 12.6 . The wo resistors R are in parallel and have equivalent resistance R/?. The circuit is equivalent to a) V, = 6.00 A)(12.0.0)- (5.00 A\(18.00)- 20.0V =F, V,—¥, =20.0V +90.0V + 60.0 V=170.0V PL-¥, =F =16.0V X—¥V,,=170.0 V so X = 186.0 V.with the upper terminal + b) £ = (6.0 V)/(8.00 2) = 2.004 ‘The junction rule applied to point a gives £, +f, = 5.00 A,so£, = 3.00A. The current through the 200.0 V battery is in the direction from the — to the + terminal, as shown in the diagram. ¢} 200.0 V— £,(2/2)=170.0V (3.00 A)(R/2) = 30.0 V so R= 20.02 2 26.66: For three identical resistors in series, P. -— If they are now in parallel over the row Itage, P, = —— = same voltage, P, RRR aR =9P, =9(27 W)= 243 W. 26.67: B= e'/R so R, =24/P P,=07/R, 50 R, =6"/P, a) When the resistors are connected in parallel to the emf, the voltage across each resistor is ¢ and the power dissipated by each resistor is the same as if only the one resistor were comnected. P,, = FR +P, b) When the resistors are connected in series the equivalent resistance is R= R +R, _# s_BP, OR+R &/R+e/P, B+P, Por 26.68: a) Tgnoring the capacitor for the moment, the equivalent resistance of the two parallel resistors is ee R, 6000 3008 60007 In the absence of the capacitor, the total current in the circuit (the current through the 8.00 Q resistor) would be po £-_80V___a nna R 8.009+2.000 of which 2/3, or 2.80 A, would go through the 3.00 @ resistor and 1/3, or 1.40 A, would go through the 6.00 @ resistor. Since the current through the capacitor is given by r ett, at the instant ¢=0 the circuit behaves as through the capacitor were not present, so the currents through the various resistors are as calculated above. b) Once the capacitor is fully charged, no current flows through that part of the circuit. The 8.00.Q and the 6.00 © resistors are now in series, and the current through them is i= 6/R = (42.0 V)/(8.00 © +600) =3.00 A. The voltage drop across both the 6.00.0 resistor and the capacitor is thus Y = iR = (3.00 A)(6.00.2) = 18.0 V. (There is no current through the 3.00 resistor and so no voltage drop across it.) The change on the capacitor is Q= CV = (4.00% 10" farad)(18.0 V) = 7.210 26.69: a) When the switch is open, only the outer resistances have current through them. So the equivalent resistance of them is: {1 1 ¥ _36.0V a = + +] =4.505f=——= 6Q43Q 30460 R, 4500 =8.00A Dhs= (38.00 4) 3.00 -(48.00 a) (6.00.2) = -12.0 V. b) If the switch is closed, the circuit geometry and resistance ratios become identical to that of Problem 26.60 and the same analysis can be carried out. However, we can also use symmetry to infer the following: Log, = BLjg, ADF Legge, = 42). From the left loop as in Problem 26.60: 1 2 36v-(F0 6 Q) =f, Q) = 0 Beg = 5.14 AM Leen = lg = TIAL 3 & 36.0V = 4.200 2 & (ce) Fatey = gia sa = G20 =8.57A> RK, £ SSTA ‘atery 26.70: a) With an open switch: V,, = ¢= 18.0 V, since equilibrium has been reached. b) Point “a” is at a higher potential since it is directly connected to the positive terminal of the battery. c) When the switch is closed: 18.0 V = 2(6.002+ 3.00 2) => F= 2.00 A = Y, = (2.00 A)(3.00 2) = 6.00 V. d) Initially the capacitor’s charges were: Q, =C¥ = (3.00% 10 FVI8.0 V)=5.40x10" C. Q, =CV = (6.00x10* FV18.0 V) =1.08 x10" C. After the switch is closed: Q, =CV = (3.0010 F)U8.0 V-12.0 V)=1.80x10" C. Q, =C¥ =(6.00x10* F)(18.0 V- 6.0 V)=7.20x10" C. So both capacitors lose 3.60x10 C. 26.71: a) With an open switch: Q, =C,,V = (2.00%10" F(18.0 V) = 3.60107 C, Also, there is a current in the left branch: 18.0 V f=—__—____= 2.00 A. 6.002+3.002 _3.6x10° C "60x10" F b) Point “b” is at the higher potential. c) If the switch is closed: ¥, =¥, = (2.00 A)G.00 Q)= 6.00. d) New charges are: Q, = C¥ = (3.00 »10% F)(6.0 V)=1.80x10° ¢ Q; =C¥ =(6.00% 10° FYC-12.0 V)=-7.20x 10" C. = AQ, = 43.6010 C— (1.80 x10 C)=+1.80x 107 C. => AQ, = —3.60 x10% C— (-7.20x 10% C)=+4+3.60%10% C. So the total charge flowing through the switch is 5.40 x10 C. 80, Vg =Veye Ven = — (2.0 A)(6.0 2) =-6.00V. 26.72: The current for full-scale deflection is 0.02 A. From the circuit we can derive three equations: (@ (R, +R, + R,)(0.100 A — 0.02 A) =48.0.2(0.02 A) DR, +R, +R, =12.00. (i) (R, + R,)0.00 A — 0.02 A) = (48.0 + R,)(0.02 A) => R +R, — 0.02042, = 09800. (ii) R, 10.0 A - 0.02 A) = (48.02 + R, + R,)(0.02 A) => R, —0.002R, — 0.002R, = 0.096 2. From (i) and (ii) = R, =10.8Q From (ii) and (iii) R, = 1.08 Q. And so => R, = 0.12 26.73: From the 3-V range: (1.00 x 10% A)(40.0.2+ R,) = 3.00 V= R, = 2960 2 = Raa = 3000 2. From the 15-V range: (1.00107 A)(40.04R, + R,)=15.0 V= R, =120002= Rican From the 150-V range: (1.00107 A)(40.02+ 2, +R, +R,)=150 V=> R, = 135,000 2 = Rega = 150 KO = 150002. ower -L 26.74: a) R,=100k0+{— 1, 1 =140kQ (200kQ 50kQ 7 MOON opewtoria. 140kQ 4 1,3) livav. 200k2 =S0kQ b) If ¥, = 5.0010" ©, then we carry out the same calculations as above to find Reg = 292KO. = £=1.37%x10 A = Pggya, = 263 V. c) If V, =, then we find 8, = 300kQ> [= 1.33x107 A = Vigan = 266 V. = Foogug = £ R= (2.86 «107 af 26.78: P= OV _ 57-190 y — LO W80kO GOkQ+ R) GOKQ+ R) = (8 VY30KQ + R) = (10 V3BOKQS R=18.5kQ, 68 Vv. 26.76: a) V =IR+IR, > R=4-R,. The tme resistance R is always less than the ze reading because in the circuit the ammetet’s resistance causes the current to be less then it should. Thus the smaller current requires the resistance R to be calculated larger than it should be. b) P=44 25 R= 7% =e. Now the current measured is greater than that through the resistor, so R=V'/J, is always greater than V’/ I. c) @: P=PR=PUV/I-R,)=WIR,. (b) P=¥?/R=VU-V/R,)=-V7/R,. 26.77: a) When the bridge is balanced, no current flows through the galvanometer: (P+X) (N+M) (PEX4+N4M) 9 (PHX4N+M) => N(P+X)=P(N+M) = NX = Pt axatt, Eq =0=9 Vy =Vp > NE yy = Ply > N yx = B50Q)33.48.2) 15.002 (b) = 18970. 26.78: In order for the second galvanometer to give the same full-scale deflection and to have the same resistance as the first, we need two additional resistances as shown below. So: (3.6 ZA)(38.0 2) = (1.496 MA)R, => R, = 91.4 mO And for the total resistance to be 65 Q: 1 65=R, + =e 0.0914 2 “1 aa) SR, =64.90 AT=B. js [J« AL-LSwA 26.79: a) P= 2” __oa (224. 2+589.Q) = Panga, = (0.111 AY(224 Q) = 24.9 V. = Fora = (0.111 A)(S89 Q) = 65.4 V. 90 ») roy tiyt 58904 Gb+ ae and Vg = &— Reese, (90 VX589 2) S89Q4 >BRV=9V- 1 sfti_) -usasr,=3840 R, 24a c) If the voltmeter is connected over the 589-2 resistor, then: 1 = 22404 eva” 589.2 ot wa) =7352 90 T= yen ODA Ty + Lapa als0 38741, = 58990 0.106 A > Voge, = Lyeyq R= (0.106 ANS89Q) = 62.4 V. d) No. From the equation in part (b) one can see that any voltmeter with finite resistance R, placed in parallel with any other resistance will always decrease the measured voltage. 2 2 j 26.80: a) (i) P,= = 922%) _az3g0w iy 2, -Z = a@’) _ 4 9, R 4.262 a 2 dC 120 V 4262 b) After a long time, = 0=> P, =0, P, =0, P, =0. (i) PB, =f = 020-V) = 3380 W. °) 1 120V 2° 24.26.) => P, = R= (14.08 A)’ (4.26 Q) = 845 W. P, iQ) _ie (4.08 A\120V) _ oye yy c 2 2 2 And P, = af = (120 V)(14.08 A) = 1690 W. When g= 50, set <055%= = 14.08 A. 26.81; a) If the given capacitor was fully charged for the given emf, Q.., =CY = (3.410 F)Q80 V) =6.12 10" C. Since ithas more charge than this after it was connecied, this tells us the capacitor is discharging and so the current must be flowing toward the negative plate. The capacitor started with more charge than was “allowed” for the given emf. Let O@=0)=Q, and QO =~) =Q,. For all t, O(4) = (Qy —O, Je“! +O, Weare given Q at some time t= 7;Q(¢=7)=8.15%10~C and from above Q, = 6.12 10™C. The current 1(¢) = 20 = G2 64° at e= 7, O(T)= (Q)-, e7*" + Q,.So the current att =T is (7) = 8 (eI *°) = OOP Thus (7) = Biers * = 8.2410 A (toward the negative plate). b) As time goes on, the capacitor will discharge to 6.12 x 10~ € as calculated above. 26.82: For a charged capacitor, connected into a circuit: Iy= 5 Q, = £,RC = (0.620 A\(S.88 kQYS.55 x 107 B) = 3.12 x10 C. 110V 26.83: 6 =1)R> R=2 = =1.69 10° QS Ip 65%10°A c= 2285 _3.67x10° F. R 1.69x10°Q 2 . 26.84: a) v= 2. eae. (4.6210 F) b) B =13Rr={ 2 p= 0.00817" agg. oe lac (850 QY4.62 x 10° F)? . ; , = 1808 W. 26.85: a) We will say that a capacitor is discharged if its charge is less than that of one electron, The time this takes is then given by: q=Qe’™ >t=RCM(Q, /e) => #= (6.7 x10° Q\9.2 107 F) In (7.0x10 C/1.6 x10" C) =19.36s, or 31.4 time constants. b) Asshown in (a), t= tIn (Q, /q), and so the number of time constants required to discharge the capacitor is independent of R and C, and depends only on the initial charge. 26.86: a) The equivalent capacitance and time constant are: =! cx=| 1 yt } = 2.00uE = r= R,,,C., = (6.00 £2)(2.00 siF) =1.20%10% 5 oe, 3 yh One b) After £=1.20x107 s,.g=0,0-e #5) =C_e(l-e M3) ¢ xii 2 ae Bang (#65) _ COMPIZ) g _ 64) = 5.06V. Cog 3.0 uF « . 2 287: a) Ey =| P, d= d= = Jew d=e7cM= 3 3 ° ° fe b) By = [2 de [PR ar~ = femne date > 3 3 2 Or FE hee Eoort we 2 2 ) One half of the energy is stored in the capacitor, regardless of the sizes of the resistor. c) U= Hy 2 Bo rss: 1 -- Lae p_ stp - Lr, pene = B= Qo 5 fer’ ae RC RC RC? Qo RC 2 “RC? 2 SoU oe ™ 26.89: a) Using Kirchhoff’s Rules on the circuit we find: Leftloop: 92-140£, — 2107, + 5-0 = 147 — 1404, — 2102, Rightloop: 57-35, — 210Z, + $5=0= 112— 210f, — 352, Currents: Sl, -1,+1,=0. Solving for the three currents we have: 1,=0300A, £, =0.500A, J, = 0.200. b) Leaving only the 92-V battery in the circuit: Left loop: 92-1402, — 210, =0. Right loop: —351,— 2102, =0. Currents: L-h+h=0. Solving for the three currents: I,=0.S541A, £,=0.077 A, I, =-0.464 A. c) Leaving only the 57-V battery in the circui Left loop: 140Z, +2102, = 0. Right loop: 57 — 352, — 2102, Currents: L-1,+1,=0. Solving for the three currents: £,=-U.287 A, fy, =UIYLA, £5 = 0480 A. d) Leaving only the 55-V baitery in the circuit Leftloop: 55-1404, — 2102, Right loop: 55-351, — 2102, Currents: Tibi Solving for the three currents: £=0046A, 1, =0.231A, J, =0.185A. e) If we sum the currents from the previous three parts we find: £,=0300A, 1, =0.500A, Z,=0.200 A, just as in part (a). £) Changing the 57-V battery for an 80-V batiery just affects the calculation in part (c). Tk changes to: Left loop: 1402, + 2102, = 0. Right loop: 80~ 352, — 2102, = 0. Currents: L-1,+1,=0. Solving for the three currents: Z,=-O403A, £, =0.269A, I, = 0.672 A. So the total current for the fulll circuit is the sum of (b), (d) and (£) above: Z=O184A, £, =0.576A, I, = 0.392 A. 26.90: a) Fully charged: Q=C¥ =(10.0x10-” F000 V)= 1.00 10% C. -Y, b) 4-22 -4_ 4 5 we (é ~ le * where C’a1ic. RR RC RRC c) We need a resistance such that the current will be greater than 1 A for longer than 200 ps. 1 4 __roass = 1000 2)-1.0720 a Hfro90— Lode )e nel 1.10.0 x10" F) 3 1.0x10~ a = 18.3R—-RInR-18x10" =0. Solving for R numerically we find 7.15 x10° Q< R<7.01 x10" Q If the resistance is too small, then the capacitor discharges too quickly, and if the resistance is too large, the current is not large enough. 26.91: ‘We can re-draw the circuit as shown below: Ri J] & & Ry | : -t =a=2n+(+2) =2R, +e, p?_oRR, -2RR, =0. R, Re R+R, => R,=R, +4R'+2RR, but R, 90S R,=R 44K 42RR. 26.92: Let current J enter at aand exit at b. At a there are three equivalent branches, so current is 7/3 in each. Atthe next junction point there are ovo equivalent branches so each gets current J/6. Then at & there are three equivalent branches with current 1/3 in each. The voltage drop from a to b then is V=(4)R+ (Z)R+(Z)R=2IR. This nust be the same as V = ZR, , so &, nok a a7 26.93: a) The circuit can be redrawn as follows: RB Then ¥, =¥, 2 —=¥,, 1 and x, = Bee +R, 2R/R, +1 aR +R, 2R(R + &) RR But £ b) Recall Ys d+) 1 3V)= A d+ A) 202+ V3) _ iRT ORR If 8 =R, 9 R,=R + VR +2RR, = 2, (1+ V3) and f 1+ v3 2.73. So, for the mth segment to have 1% of the original voltage, we need: 1 1 a+ fy da27 c) R,=R, 44k 42R,R, = R, = 6400 2 4+ (6400 2)? + 2(6400 ©) (8.0 x 10° Q) = 3.2 «10° @ = p= MSAD 2)3.2%10' 048.010! 2) B.2x10° Q) 6.0% 10° Q) <001l>n=4:¥, = 0.005%. 0x10 d) Along a length of 2.0 mm of axon, there are 2000 segments each 1.0 ym long. The voltage therefore attenuates by: % Foon 1 4 = =p Ato = ga 04, a+ py" " % d+4.0x107)"* Veco 2) If 8, = 3.310" QR, =2.1x10° Q and f= 62x10". Yoon %, 0462x107) = = 0.88. F = gi x B(-1.24 x10 C(-3.85 x 10' m/s )1.40 NG xD =~ (6.68 «107 Nk. b) F=gixB => F =(-1.24 10 C).40 T)[(—3.85 104 mn/ay(f x B) + (4.19 x 10% m/s\? » B)] => F =(6.68x 104 N)i+(7.27x10~ Nj. 27.2: Need a force from the magnetic field to balance the downward gravitational force. Its magnitude is: -4 2 pB=mg = B= BE = 0.95x10 9980 m/s") -191T. qv (2.5010 C)(4.00x 10" m/s) ‘The right-hand rule requires the magnetic field to be to the east, since the velocity is northward, the charge is negative, and the force is upwards. 27.3: By the right-hand rule, the charge is positive. Zz Wd: Fa maaqex Ba 228 ™ . 8 4 8 . jw 22x10 ©)G.0 «10 m/s}0.68 DUD __ ©3530 nys*yh. 1.81%107 ke 27.5: See figure on next page. Let Fy = qvB, then: F, =F, in the —k direction F,= F, inthe +} direction F,=0, since Band velocity are parallel F,=F, sin 45° in the — j direction FL = F, inthe — (}+ &) direction 27.6: a) The smallest possible acceleration is zero, when the motion is parallel to the magnetic field. The greatest acceleration is when the velocity and magnetic field are at right angles: ais . 2 = 2B _ G61? CH2-50 10% ms 74107 TD) _ 9 95,19! ays? m (9.11x10™ kg) b) If a= 25x 10%m/s?) - 2280 6 _, sin § = 0.25 > =14.8. m EF 4.60 x10" N jq[Bsing (1.6x10" C)G.5x10% T) sin 60° Bsin d v= = 9.49 10° m/s. 278: a) F = qvx B=98, lv, xB +v,GxB+v,Ex b=, D+v,O) Set this equal to the given value of F to obtain: F, (7.40 x10" N) ~ 60%107 CX pa 10H = — 60x10 N) _ 48.6 ms. (=5.60x10 €)(=1.25T) b) The value of v, is indeterminate. F, + £42 =0;0=90. — 4B, @B, 6) ¥-F =F, +y,F, +v.F, = 27.9: F= qv »B,¥=v,} with, =—3.80 x10 m/s F, =+7.60x107 N, F, =O,and F, =-5.20x10°N q,B, — v,B,) = 4v,B, B, =F, qv, = (7.6010 N)/([780x 10% C)(—3.80 x10° m/s)] =— 0.256 T F, =q(v,B, ~ v,B,)=0, which is consistent with Fas given in the problem. No force componentalong the direction of the velocity. F,=q,B, — v,B,)=—4v,B, B,=-E,/qv,=-0.175T b)_B, is not determined. No force dne to this component of B along v: measurement of the force tells us nothing about B,. o) BF =B,F,+B,F, + BF, =(—0.175 T)(+7.60%10° N)+ (0.256 T)(—5.20x1077 NH B - F =0; Band Fare perpendicular (angle is 90°) 27.10: a) The total flux must be zero, so the flux through the remaining surfaces must be 0.120 Wb. b) ‘The shape of the surface is unimportant, just that it is closed. WA: a) ©, = B.A = (0.230 T)x(0.065 m)* = 3.05 x 10% Wh. b) &, = B- A=(0.230T)x(0.065 m)" cos 53.1°= 1.83%10° Wb. ) ®,=Osince BL 27.12: a) ©, (abcd) =B- A= 0. b) ©, Gefe) = B- A= —(0.128T\(0.300 m)(0.300 m) = - 0.0115 Wo, °) @ s(aefd) = Bd = Boos 6-2 (0.128 T)(0.500 m)(0.300 m) = +0.0115 Wb. d) The net flux through the rest of the surfaces is zero since they are parallel to the x- axis so the total flux is the sum of all parts above, which is zero. 27.13: a) B = [(6 — yy’ )]j and we can calculate the flux through each surface. Note that there is no flux through any surfaces parallel to the y-axis. Thus, the total flux through the closed surface is: ® , (abe) = B+ A = ([-(0.300T - 0)] + [0.300 T - (2.00 T/m? )(0.300 m)?}) x 710.400 m)(0.300 m) =— 0.0103 Wb. b) The student’s claim is implausible since it would require the existence of a magnetic monopole to result in a net non-zero flux through the closed surface. 273429) p=m=m| S22) - RaB = (4.68 x10 m)(6.4% 10 C\(1.65 T) =4.94 x10 ke m/s. b) L= Rp = R%gB = (4.68107 m)'(6.4 x10" CV1.65T) = 2.31% 10 ke m/s. my _ (9.114107 kg\(1.41 x 10% m/s) JR 60x 10” Cj.0500 m) ‘The direction of the magnetic field is into the page (the charge is negative). b) The time co complete half a circle is just the distance traveled divided by the velocity: 27.15: a) B= =1.61x10% T. pe Peg ER 200500 my 11074 yoy 1Alx10%m/s 67 x10" ke)(1.41x 10° qaé:a) B=™ = 0.672107 beg) (1-41 10° nyo) @R (1.60 x10 C\(0.03500 m) The direction of the magnetic field is out of the page (the charge is positive), b) The time to complete half a circle is unchanged: ¢=1.11%107 5. =0.294T QWAT: K+, =Ky +U; U,=K,=0, so K, =U,; Lim = er ak ¥ =1.2x10' m/s = (602x107 cy |_———_— mr (3.34.%10- ky)(1.0 «1077 wn) ee 3 b) )\F =magives gvB = m*/r _ omy _ 3.34107 ke V1.2 107 avs) _ Bia TY _ Bee BIE ee eS) ono ar (1.602 x10” C\(2.50 m) 27.18: a) F =qvB sind F 0.00320 x 10° N “qvsin@ 8(1.60%10" C)(500, 000 m/s) sin 90° B= 5,00. Ifthe angle@ is less than 90°, a larger field is needed to produce the same force. The direction of the field must be toward the south so that » x Bcan be downward. b) F=qBsind F 4.6010 N @B sin® (1.60%10" Cy2.10T) sin 90° y= 1.37% 10" m/s.If @ is less than 90° , the speed would have to be larger to have the same force. The force is upward, so v x B must be downward since the electron is negative, so the velocity must be toward the south. 27.19: q = (4.00 x 10° y -1.602 x10" C)= 6.408 x10 € speed at bottom of shait: 4 mv? = mgy,v = /2gy = 49.5 m/s » is downward and Bis west, so vx Bisnorth. Since q <0, F is south. F = qB sin 9 = (6.408 x 10™ C)(49.5 m/s (0.250 T) sin 90° = 7.93x 10" N my 27.20: ae @ @B ya GBR _ 301.60 10°? €)0.250 TC? m) m 12(1.67x107" ke) y= 2.84 10° m/s Since # x B is to the left but the charges are bent to the right, they must be negative. D) Fea, = mg = 12(1.67x 10” kgy(9.80m/s” )=1.96x 10 N ue = VB = 301.6 x10" C)(2.84 x 10° ma/s (0.250 T) =3.41x107N Since Fay, ¥10!7 x Fi. we can safely neglect gravity. Fane c) The speed does not change since the magnetic force is perpendicular to the velocity and therefore does not do work on the particles. _@RB _ (1.60x10" C)(6.96x10 my2.50T) nm (334x107? kg) _D_#R_ (6.96 x10"m) yy 884% 10° m/s my? _(3.34.x1077 kp)(8.34 «10° m/s)" 2q 21.60 x10" C) 27.21: a) v = 8.34 x 10° m/s. b) ¢ =2.62 x10" s. 3) fom =q¥>V= =7260V. 31 dads Ra = OALAO™ begV2.8210% m/s) 1 9904, gB (160x109 C)(0.0877 T) 2: 27.93: a) B= : lal (1.60 10" C) This is about 2.4 times the greatest magnitude yet obtained on earth. b) Protons have a greater mass than the electrons, so a greater magnetic field would be required to accelerate them with the same frequency, so there would be no advantage in using them. 31 x10! OM xI Oe Pe G00 10 CHE) _ io 27.24: The initial velocity is all in the y-direction, and we want the pitch to equal the radius of curvature my, > d,=v,F=—= @B Qn _ Dam o @B But r= a 2a Oy Bg B Qn = tan@=> @=81.0°. 27.25: a) The radius of the path is unaffected, but the pitch of the helix varies with time as the proton is accelerated in the x-direction. by ran (1.67 x10" ke) w @B (1.60x10" C)O.500T) Fo q@E_ (1.6%107? C)(2.00 «104 V/m) ge, MON * See a) “nm om 1.67«10 ke =131«107 3,t=T/2,and =1.9210'? m/s*. 4 92x 10" m/s’ )(6.56 x 10° 5)? d. ave afl = (Sy IO NG 56410 8 5 >¢,=001m. FOO 27.26: Lmy? = qh 5 y= PEE = PES OOO) _ 19,19 ms, 2 m (1.16107 kp) my (1.16% 10 ke)(7.79 x 104 m/s) = R= OE 7.81107 mo. @B (1.6010? C)(0.723T) 9| 46x10 OMMO0vIE pove ay 20.6 x10 C) 2.0 x10 V) Vom (9.11x10™ kg) =2.65 x10" m/s. 31 2 = BY _ O11 10"! kg)@.65 10" mys) _ 9 ge gp jd (1.60 x10" €)(0.180 m) a7.at: Any? 2 27.28: a) v= E/B =(1.56« 10" V/m )/(4.62 «107 T) = 3.38 x 10° m/s. b) E & pa BY GALAO™ kg).38 x10 m/s) \qlB (.60x 10° C)(4.62 x 10° T) => R=417%107 m _ 2mm QR _2x(4.1T1O™ mh) 474 19g By G.38 x 10% ays) 27.29: a) F, =F, 30 |gvB=|E; 8 = E/v=0.10T Forces balance for either sign of g. ) E=V/d so v= B/B=V/aB smallest v: 120V largest V7, smallest B, vj, — ————“—____ = 2.1» 10* m/s (0.0325 m)(0.180T) largest ¥: smallest V7, largest B, v, 560V =3.2x10° m/s ‘=a (C0325 my(0.054 T) 27.30: To pass undeflected in both cases, £ = vB = (5.85 x 10° m/s)(1.35 T)= 7898 N/C. a) If g= 0.640107 C, the electric field direction is given by -— (7x (- &)) since it must point in the opposite direction to the magnetic force. b) If ¢=-0.320x 107 ©, the electric field direction is given by ((—f) x (A) since it must point in the same direction as the magnetic force, which has swapped from part (a). The electric force will now point opposite to the magnetic force for this negative charge using Fe = gE. mE RgB? _ (0.310 my1.60 x10 C0.540 T)* qBe E (1.12 «10° V/m) =1.29 x10 kg 27.31: R=" = qB

You might also like